Vous êtes sur la page 1sur 104

Certificate Mathematics in Action Full Solutions 4B

9 More about Trigonometry (II)


• • • • • • • • • • • • • • • • • • • • • • • • • • • • • • • • • • • • •

Activity A  B  C  180
3. ( sum of △)
A  76  51  180
Activity 9.1 (p. 202) A  53
1. (a) Yes
(b) YP

(c) Slope of PF =
FD , slope of PX = XY ,
DP YP
slope of PE =
EC
CP
2. PX. Since FD = XY = EC and YP is the shortest in
length among line segments between AB and DC,

therefore
XY has the greatest value.
YP

Follow-up Exercise

p. 168

Sides Angle Area


1. a = 4 cm, b = 6 cm C = 30 6 cm2
2. a = 5 cm, b = 8 cm C = 45 14.1 cm2
3. b = 7 cm, c = 9 cm A = 102 30.8 cm2
4. a = 7 cm, c = 9 cm B = 110 29.6 cm2
5. a = 12 cm, c = 9 cm B = 90 54 cm2

p. 175
1. By the sine formula,
AB AC

sin C sin B
x cm 8 cm

sin 30 sin 43
8 sin 30
x
sin 43
 5.87 (cor. to 3 sig. fig.)

A  B  C  180
2. ( sum of △)
70  B  54  180
B  56
By the sine formula,
AB AC

sin C sin B
x cm 7 cm

sin 54 sin 56 
7 sin 54
x
sin 56
 6.83 (cor. to 3 sig. fig.)

1
9 Basic Properties of CirclesMore about Trigonometry (II)

By the sine formula,


BC AC

sin A sin B
23 cm x cm

sin 53 sin 76
23 sin 76
x
sin 53
 27.9 (cor. to 3 sig. fig.)

A  B  C  180
4. 48  B  62  180 ( sum of △)
B  70
By the sine formula,
a AB

sin A sin C
a 5 cm

sin 48 sin 62
5 sin 48
a cm
sin 62
 4.21 cm (cor. to 3 sig. fig.)
b AB

sin B sin C
b 5 cm

sin 70 sin 62
5 sin 70
b cm
sin 62
 5.32 cm (cor. to 3 sig. fig.)

A  B  C  180
5. 28  115   C  180 ( sum of △)
C  37
By the sine formula,
a AB

sin A sin C
a 9 cm

sin 28 sin 37
9 sin 28
a cm
sin 37
 7.02 cm (cor. to 3 sig. fig.)
b AB

sin B sin C
b 9 cm

sin 115  sin 37
9 sin 115 
b cm
sin 37
 13.6 cm (cor. to 3 sig. fig.)

A  B  C  180
6. 60  50  C  180 ( sum of △)
C  70

2
Certificate Mathematics in Action Full Solutions 4B

By the sine formula, c a


b BC 
 sin C sin A
sin B sin A c 7 cm
b 10 cm 
 sin 18.265 sin 43
sin 50 sin 60
7 sin 18.265
10 sin 50 c cm
b  cm sin 43
sin 60
 3.22 cm (cor. to 3 sig. fig.)
 8.85 cm (cor. to 3 sig. fig.)

c BC

sin C sin A
c 10 cm

sin 70 sin 60
10 sin 70
c cm
sin 60
 10.9 cm (cor. to 3 sig. fig.)

p. 178
1. By the sine formula,
a b

sin A sin B
7 cm 9 cm

sin 43 sin B
9 sin 43
sin B 
7
 0.8769
B  61.3 (cor. to 3 sig. fig.) or
119  (cor. to 3 sig. fig.)
When B = 61.3,
A  B  C  180 (  sum of )
43  61.265  C  180
C  75.7 (cor. to 3 sig. fig.)
By the sine formula,
c a

sin C sin A
c 7 cm

sin 75.735 sin 43
7 sin 75.735
c  cm
sin 43
 9.95 cm (cor. to 3 sig. fig.)
When B = 119,
A  B  C  180
43  118 .73  C  180
C  18.3

( sum of )

(cor. to 3 sig. fig.)


By the sine formula,

3
9 Basic Properties of CirclesMore about Trigonometry (II)

2. By the sine formula, c a



sin C sin A
a b
 c 6 cm
sin A sin B 
sin 60 sin 30
10 cm 8 cm
 6 sin 60
sin 35 sin B c  cm
sin 30
8 sin 35
sin B   10.4 cm (cor. to 3 sig. fig.)
10
 0.4589
B  27.3 (cor. to 3 sig. fig.) or 152.7 (rejected)
A  B  C  180 ( sum of )
35  27.314  C  180
C  118 (cor. to 3 sig. fig.)
By the sine formula,
c a

sin C sin A
c 10 cm

sin 117.69 sin 35
10 sin 117.69
c cm
sin 35
 15.4 cm (cor. to 3 sig. fig.)

3. By the sine formula,


a b

sin A sin B
6 cm 8 cm

sin 130 sin B
8 sin 130
sin B 
6
 1.02 (cor. to 3 sig. fig.)
∵ sin B  1 for all values of B.
∴ There are no solutions for B.

4. By the sine formula,


a b

sin A sin B
6 cm 12 cm

sin 30 sin B
12 sin 30
sin B 
6
1
∴ B  90
A  B  C  180
( sum of △)
30  90  C  180
C  60
By the sine formula,

4
Certificate Mathematics in Action Full Solutions 4B

p. 185 C  180  A  B
1. (a) By the cosine formula,  180  52.14  47.15
 81 (cor. to the nearest degree)
b 2  a 2  c 2  2ac cos B
x  17 2  20 2  2(17)( 20) cos 125 4. By the cosine formula,
 32.8 (cor. to 3 sig. fig.)
(b) By the cosine formula, c 2  a 2  b 2  2ab cos C
c  40 2  552  2( 40)( 55) cos 100 cm
c 2
 a 2
 b 2
 2ab cos C  73.4 cm (cor. to 1 d.p.)
x  25  22
2 2
 2( 25)( 22) cos 42
a 2  c 2  b2
 17.1 (cor. to 3 sig. fig.) cos B 
2ac
402  73.4102  552
2. (a) By the cosine formula, 
2( 40)( 73.410)
a 2  b2  c 2 B  47.5 (cor. to 1 d.p.)
cos  
2ab A  180  100  47.547
18  14 2  112
2
 32.5 (cor. to 1 d.p.)

2(18)(14)
133 p. 189

168 a  b  c , where a = 14 m, b = 8 m and c = 10 m.
1. Let s 
  37.7 (cor. to the nearest 0.1) 2
14  8  10
(b) By the cosine formula, s  m
2

a 2  c 2  b2  16 m
cos  
2ac s  a  (16  14) m  2 m
182  17 2  312 s  b  (16  8) m  8 m

2(18)(17) s  c  (16  10) m  6 m
29 ∴ Area of △ABC
 
51
 s ( s  a )( s  b)( s  c )
  124.7 (cor. to the nearest 0.1)
 16  2  8  6 m 2

3. By the cosine formula,  39.2 m 2 (cor. to 3 sig. fig.)

b2  c 2  a 2
cos A  a  b  c , where a = 33 m, b = 29 m and c = 24 m.
2bc 2. Let s
26  352  282
2 2
 32  29  24
2( 26)( 35)
∴ s  2
m
1117
  43 m
1820
A  52 (cor. to the nearest degree) s  a  ( 43  33) m  10 m
s  b  (43  29) m  14 m
a 2  c 2  b2 s  c  ( 43  24) m  19 m
cos B 
2ac ∴ Area of △ABC
28  352  262
2
  s ( s  a )( s  b)( s  c )
2( 28)(35)
1333  43  10  14  19 m 2
  338 m 2 (cor. to 3 sig. fig.)
1960
B  47 (cor. to the nearest degree)
∴ a  b  c , where a = 26 m, b = 37 m and c = 21 m.
3. Let s 
2

5
9 Basic Properties of CirclesMore about Trigonometry (II)

26  37  21
∴ s  m
2
 42 m
s  a  ( 42  26) m  16 m
∴ The required compass bearing is N80E.
s  b  (42  37) m  5 m (b)
s  c  ( 42  21) m  21 m
∴ Area of △ABC

 s ( s  a )( s  b )( s  c )
 42  16  5  21 m 2
 266 m 2 (cor. to 3 sig. fig.)

180  120 = 60


a  b  c , where a = 17 m, b = 27 m and c = 17 m.
4. Let s  ∴ The required compass bearing is S60E.
2 (c)
17  27  17
∴ s  m
2
 30.5 m
s  a  (30.5  17) m  13.5 m
s  b  (30.5  27) m  3.5 m
s  c  (30.5  17) m  13.5 m
∴ Area of △ABC

 s ( s  a )( s  b)( s  c )
 30.5  13.5  3.5  13.5 m 2
 139 m 2 (cor. to 3 sig. fig.)

p. 192
1.

With the
notations in the figure,
ABD  BAC  ACB
(a)
 20  35
 55
∴ The angle of elevation of A from B is 55.
(b) EAC  ACB
 35
EAB  EAC  BAC
 35  20
 55
∴ The angle of depression of B from A is 55.
(c) EAC  35
∴ The angle of depression of C from A is 35.

2. (a)

6
Certificate Mathematics in Action Full Solutions 4B

250  180 = 70


∴ The required compass bearing is S70W.
(d)

360  330 = 30


∴ The required compass bearing is N30W.

3. (a)

∴ The required true bearing is 048.


(b)

180  57 =
123
∴ The required true bearing is 123
.
(c)

180  62 = 242


∴ The required true bearing is 242.
(d)

360  28 =
332
∴ The required true bearing is 332
.

7
9 Basic Properties of CirclesMore about Trigonometry (II)

4. By the sine formula,


CB 200 m

sin 35 sin 30
200 sin 35
CB  m
sin 30 (alt. s, AD // EC)
CBD  ECB
 65
In △CBD,
CD
sin CBD 
CB
With the notations in the figure, CD  CB sin CBD
DBA  BAF 200 sin 35
(a) (alt. s, DG // AF)    sin 65  m
 30  sin 30 
∴ The compass bearing of A from B is S30E.  207.9 m (cor. to 4 sig. fig.)
CBD  CBA  ABD
(b) ∴ The height of the building is 207.9 m.
 35  30
 65 p. 201
GBC  180  CBD 1. (a) ∵ HAD is the angle between planes ABGH
and ABCD.
 180  65 ∴ The angle between planes ABGH and
 115  ABCD is 40.
∴ The true bearing of C from B is 115. (b) ∵ AHD is the angle between planes ABGH
and DCGH.
(c) BCH  CBD (alt. s, IH // DG) AHD  HAD  ADH  180 (
 65
AHD  40  90  180
∴ The compass bearing of B from C is N65W.
ACI  FAC AHD  50
(d) (alt. s, IH // AF) sum of △)
 50
HCA  180  ACI ∴ The angle between planes ABGH and
Reflex DCGH is 50.
 180  50
(c) ∵ EAH is the angle between planes ABGH
 230 and ABFE.
∴ The true bearing of A from C is 230. EAH  AHD (alt. s, EA // HD)
p. 195  50
1. By the cosine formula, ∴ The angle between planes ABGH and
ABFE is 50.
QR 2  PQ 2  PR 2  2( PQ )( PR ) cos QPR (d) ∵ EHA is the angle between planes ABGH
and EFGH.
QR  202  152  2( 20)(15) cos 55 km EHA  HAD
(alt. s, EA // HD)
 17 km (cor. to the nearest km)  40
∴ The distance between town Q and town R is 17 km. ∴ The angle between planes ABGH and
EFGH is 40.
2. (e)

With the notations


in the figure,
ACB  ECB  ECA
With the notations in the figure,
 65  35
∵ EJA is the angle between planes ABGH
 30 and EFCD.
CAB  ECA (alt. s, AD // EC) JA = JD (property of rectangle)
 35 JAD  JDA (base s, isos. △)
 40

8
Certificate Mathematics in Action Full Solutions 4B

EJA  JAD  JDA (ext.  of BD  8 2  10 2 m (Pyth. theorem)


 40  40  164 m
 80 HD  GC  6 m
△)

∴ The angle between planes ABGH and


EFCD is 80.

2. (a) (i) ∵ VH is the height of the pyramid.


∴ H is the projection of V on plane
ABCD.
∴ HA is the projection of VA on plane
ABCD.
∴ VAH is the angle between VA and
plane ABCD.
(ii) ∵ VH is the height of the pyramid.
∴ H is the projection of V on plane
ABCD.
∴ HM is the projection of VM on plane
ABCD.
∴ VMH is the angle between VM and
plane ABCD.
(b) (i) ∵ VM  AB and HM  AB
∴ VMH is the angle between planes
VAB and ABCD.
(ii) ∵ DH  VH and AH  VH
∴ DHA is the angle between planes
VHA and VHD.

p. 208
1. (a) (i)

(ii)

The angle between HB and plane HDCG is


BHC.
CH  6 2  10 2 m (Pyth. theorem)
 136 m
Consider △BCH.

8
tan BHC 
136
BHC  34.4 (cor. to 3 sig. fig.)
∴ The angle between HB and plane
HDCG is 34.4.
(b) The angle between HB and BD is HBD.

9
9 Basic Properties of CirclesMore about Trigonometry (II)

Consider △BDH.
Let M be the mid-point of AB.
HD ∵ △VAB is an isosceles triangle.
tan HBD  ∴ VM  AB
BD
6

164
HBD  25.1 (cor. to 3 sig. fig.)
∴ The angle between HB and BD is 25.1.

2. (a)

(b)
The angle between the planes ACFD and
BCFE is ACB.
Consider △ABC.

AC 2  BC 2  AB 2
cos ACB 
2 AC  BC
28  20 2  14 2
2

2(28)( 20)
247

280
ACB  28.1 (cor. to 3 sig. fig.)
∴ The angle between the planes ACFD and
BCFE is 28.1.

3. (a) The angle between VA and plane ABCD is


VAO.
AC  212  202 cm
 29 cm
1
∴ OA  AC  14.5 cm (property
2
of rectangle)
Consider △VAO.

20
tan VAO 
14.5
VAO  54.06 (cor. to 4 sig. fig.)
∴ The angle between VA and plane ABCD is
54.06.
(b)

10
Certificate Mathematics in Action Full Solutions 4B

∵ △OAB is an isosceles triangle. TO


∴ OM  AB tan TBO 
∴ AB is the line of intersection of the planes OB
VAB and ABCD. 115 .470 m
∴ The angle between planes VAB and ABCD 
360.555 m
is VMO.
TBO  17.8 (cor. to 3 sig. fig.)
21
OM  cm ∴ The angle of elevation of T from B is
2 17.8.
 10.5 cm
Consider △VOM.

20
tan VMO 
10.5
VMO  62.30 (cor. to 4 sig. fig.)
∴ The angle between the planes VAB and
ABCD is 62.30.
(c) VO is the line of intersection of the planes VAO
and VDO.
∵ DO  VO and AO  VO
∴ The angle between the planes VAO and
VDO is DOA.
Consider △DOA.
OD  OA (property of rectangle)
 14.5 cm
By the cosine formula,

OD 2  OA2  DA2
cos DOA 
2OD  OA
14.5  14.52  212
2

2(14.5)(14.5)
41
 
841
DOA  92.79 (cor. to 4 sig. fig.)
∴ The angle between the planes VAO and
VDO is 92.79.

p. 216
1. (a) Consider △TOA.
TO
tan 30 
AO
TO  200 tan 30 m
 115 m (cor. to 3 sig. fig.)
∴ The height of the tower TO is 115 m.
(b) Consider △OAB.
∵ The man walks westwards to a point B.
∴ B is due west of A.
∴ OAB = 90
OB 2  OA2  AB 2 (Pyth. theorem)
OB  2002  3002 m
 360.555 m
Consider △TOB.

11
9 Basic Properties of CirclesMore about Trigonometry (II)

(c)
AB 2  OA2  OB 2  2(OA)(OB) cos AOB
2 2
350 2  h    h   2 h  h  cos 50
 tan 50   tan 30   tan 50  tan 30 
 1 1 2 cos 50  2
122 500    h
 tan 50 tan 30 tan 30tan 50 
2 2

122 500
h 
1 1 2 cos 50
With the notations in the figure, consider  
△OAB. tan 50 tan 30 tan 30tan 50
2 2

AB  258 (cor. to 3 sig. fig.)


tan AOB 
OA ∴ The height of the tower TO is 258 m.
300 m Exercise

200 m Exercise 9A (p. 169)
AOB  56.310 Level 1
COB  COA  AOB 1. (a) Area of △ABC
Reflex  180  56.310 1
 236 (cor. to 3 sig. fig.)    16  12  sin 74  cm 2
2 
∴ The true bearing of B from O is 236.  92.3 cm 2 (cor. to 3 sig. fig.)
2. (b) Area of △ABC

1
 
  10  8  sin 142   cm
2
2 
 24.6 cm 2 (cor. to 3 sig. fig.)
(c) BC = BA = 7 cm
Area of △ABC

1
 
  7  7  sin 43  cm
2
2 
Let h m be the height of the tower TO.
Consider △TAO.  16.7 cm 2 (cor. to 3 sig. fig.)
hm
tan 50  (d) ∵ △ABC is an equilateral triangle.
OA ∴ B = 60 and BC = AC = 4 cm
h Area of △ABC
OA  m
tan 50 1
Consider △TBO.  
  4  4  sin 60  cm
2
2 
hm
tan 30   6.93 cm 2 (cor. to 3 sig. fig.)
OB
h
OB  m 2. (a) Area of △
tan 30
With the notations in the figure, 1
AOB  90  BOC ABC   AC  BC  sin C
2
 90  40 1
 50 7.5   x  5  sin 65
2
Consider △OAB. x  3.31 (cor. to 3 sig. fig.)
(b) Area of △

1
ABC   AB  AC  sin A
2
1
29   12  x  sin 138
2
x  7.22 (cor. to 3 sig. fig.)

12
Certificate Mathematics in Action Full Solutions 4B

5.
3. (a) Area of △ 1
Area of ABC   AB  AC  sin BAC
2
1
PQR   PR  QR  sin R 1
2 32   7  11  sin BAC
2
1
15   8  5  sin  64
2 sin BAC 
  48.6 (cor. to 3 sig. fig.) or 77
BAC  56.22 or 180  56.22
131 (rejected)  56.2 (cor. to 3 sig. fig.) or
(b) Area of △
124 (cor. to 3 sig. fig.)
1
PQR   PQ  PR  sin P 6. (a)
2
1
20   7.4  12.3  sin 
2
  26.1 (cor. to 3 sig. fig.) or
154 (rejected)
Join BD.
1 Area of △ABD
ABC   AB  AC  sin BAC
4. Area of △ 2 1
1    5  7  sin 68  cm 2
55   12  10  sin BAC 2 
2 ∴ Area of ABCD
11 = 2  area of △ABD
sin BAC 
12
=  2  1  5  7  sin 68  cm 2
BAC  66.4 (rejected) or  2 
114  (cor. to 3 sig. fig.) =
32.5 cm 2 (cor. to 3 sig. fig.)

(b)

Join AC.
ADC  ABC (opp. s of // gram)
 132
Area of △ACD =
 1  8  6  sin 132  cm 2
2 
∴ Area of ABCD
= 2  area of △ACD

=  2  1  8  6  sin 132  cm 2
 2 
=
35.7 cm 2 (cor. to 3 sig. fig.)

7.

Join BD.
AB  AD (property of rhombus)
 14 cm

13
9 Basic Properties of CirclesMore about Trigonometry (II)

Area of △ABD = = area of △ACD + area of △ABC


 1  14  14  sin 48  cm 2 1
 AD  AC  sin CAD 
2  = 2
∴ Area of rhombus ABCD 1
= 2  area of △ABD  AC  AB  sin CAB
2
=  2  1  14  14  sin 48  cm 2  1  5  7  sin 28  1  7  10  sin 52  cm 2
 2  =
2 2 
= = 35.8 cm 2 (cor. to 3 sig. fig.)

11. (a) Area of the shaded segment


= area of sector OAB  area of △OAB

135 1
146 cm 2 (cor. to 3 sig. fig.)     20 2    20  20  sin 135  cm 2
 360 2 
8.
 330 cm (cor. to 3 sig. fig.)
2

(b) ∵ EA  AB  BC  CD  DE
(given)
Join AC. ∴
CBA  DAB  180 (int. s, CB //
CBA  160  180 EOA  AOB  BOC
CBA  20  COD  DOE (equal chords, equal s)
DA) 360
EOA 
Area of parallelogram ABCD
5
 2  area of ABC  72
1
 2   AB  BC  sin CBA
2

1
38 cm 2  2   14 cm  BC  sin 20
2
38
BC  cm
14 sin 20
 7.94 cm (cor. to 3 sig. fig.)

9. ∵ △ABC is an isosceles triangle.


∴ AC  AB
 6.5 cm
ACB  ABC (base s, isos. △)
 28
BAC  ABC  ACB  180 ( sum of
BAC  28  28  180
BAC  124
△)

Area of the triangle

1
  AB  AC  sin BAC
2
1
   6.5  6.5  sin 124  cm 2
2 
 17.5 cm 2 (cor. to 3 sig. fig.)

10. Area of quadrilateral ABCD

14
Certificate Mathematics in Action Full Solutions 4B

Area of the shaded segment 15. ∵ △ABC is an isosceles triangle.


= area of sector OAE  area of △OAE ∴ AC = BC = 9 cm
BAC  ABC (base s, isos. △)
72 1
    82    8  8  sin 72  cm 2  70
 360 2  ACB  BAC  ABC  180 ( sum
 9.78 cm (cor. to 3 sig. fig.)
2
ACB  70  70  180
ACB  40
12. Let AB = x cm and BC = (x + 2) cm, where x is an of △)
integer.
∵ 10 cm2 < area of △ABC < 20 cm2 DAC  ACB (alt. s, AD // BC)

 40
Area of trapezium ABCD
1
10 cm 2   AB  AC  sin ABC  20 cm 2 = area of △ABC + area of △ACD
2
1 1
10  x ( x  2) sin 30  20   AC  BC  sin ACB 
2 2
1 1
10  x ( x  2)  20  AD  AC  sin DAC
4 2
1 1
40  x ( x  2)  80    9  9  sin 40   6  9  sin 40  cm 2
∴ x must satisfy the inequality 40 < x(x + 2) < 80. 2 2 
Take x = 6,  43. 4 cm 2 (cor. to 3 sig. fig.)
x ( x  2)  6(6  2)
 48
16. Consider △ABD.
∴ The possible lengths of AB and BC are 6 cm and
8 cm respectively. (or any other reasonable BD 2  AB 2  AD 2 (Pyth. theorem)
answers)
BD  52  12 2 cm
13. ∵ The areas of △ABC and △XYZ are the same.  13 cm
∴ ∵ Area of △ABD = area of △BCD
1 1
 AB  BC  sin ABC   XY  YZ  sin XYZ 1  AB  AD  1  BD  BC  sin DBC
2 2 2 2
1 1 1 1
 3  4  sin ABC   6  2  sin XYZ  5  12   13  8  sin DBC
2 2 2 2
sin ABC  sin XYZ 15
sin DBC 
ABC = XYZ (rejected) or ABC = 180  XYZ 26
Take ABC = 30, DBC  35.23 or 180  35.23
30  180  XYZ  35.2 (cor. to 3 sig. fig.) or
XYZ  150
∴ The possible values of ABC and XYZ are 30 145 (cor. to 3 sig. fig.) (rejected)
and 150 respectively. (or any other reasonable
answers)
KC  AK
Level 2
ADB  DBC  BCD 1
 AC
14. (ext.  of △) 17. 2 (diags. of // gram)
 35  50
1
 85    10  cm
Area of △ABC 2 
= area of △BCD + area of △ABD  5 cm
1

2
 BC  BD  sin DBC  KD  BK
1 1
2
 AD  BD  sin ADB  BD (diags. of // gram)
2
 1  7  5.38  sin 35   1

 2

 cm
2    14  cm
 1  2 
 3  5.38  sin 85
2   7 cm
 18.8 cm 2 (cor. to 3 sig. fig.) Area of parallelogram ABCD
= 4  area of △CDK

15
9 Basic Properties of CirclesMore about Trigonometry (II)

1 Area of major segment BAC


 4  KC  KD  sin DKC = area of sector OACB + area of △OAB
2
1
  4   5  7  sin 70  cm 2     12 2 
230

1
 12  12  sin 130  cm 2
 2   360 2 
 65.8 cm (cor. to 3 sig. fig.)
2
 344 cm (cor. to 3 sig. fig.)
2

18. Reflex AOB  360  130 (s at a pt.) OC  CA


 230 19. 8 (given)
 cm
2
 4 cm
Area of the shaded region
= area of sector OAB  area of △OCB

45 1
    82    4  8  sin 45  cm 2
 360 2 
 13.8 cm (cor. to 3 sig. fig.)
2

20.

Join OC.
∵ OC  OA (radii)
∴ ACO  CAB (base s, isos. △)
 35
BOC  ACO  CAB (ext.  of )

 35  35
 70
Area of the shaded region
= area of sector OBC  area of △OBC

70 1
    7 2    7  7  sin 70  cm 2
 360 2 
 6.91 cm (cor. to 3 sig. fig.)
2

21. (a) Consider △ABC and △ADE.


BAC  DAE  70 (common
angle)
ABC  ADE (corr. s, BC // DE)
ACB  AED (corr. s, BC // DE)
∴ △ABC ~ △ADE (AAA)

AB AC
 (corr. sides, ~ s)
AD AE
AB AC

AB  BD AC  CE
6 cm 4 (b)

6 cm  BD 42
6 cm  BD  9 cm
BD  3 cm
Area of BDEC

16
Certificate Mathematics in Action Full Solutions 4B

= area of △ADE  area of △ABC AC BC



sin B sin A
 1  (6  3)  ( 4  2 )  sin 70  
2  x cm 7 cm
  
2
 cm

2
1
 6  4  sin 70 

sin 116  sin 28
 14.1 cm 2 (cor. to 3 sig. fig.) 7 sin 116 
x 
OP  OQ sin 28
22. (a)  BC (radii)  13.4 (cor. to 3 sig. fig.)
 10 cm
3. By the sine formula,
OD  OC (given)
1 AC AB
 CD 
(b) 2 sin B sin C
1 14.7 cm 12 cm
   10  cm sin 68

sin x
2 
 5 cm 12 sin 68
sin x 
Consider △POD. 14.7
OD x  49.2 (cor. to 3 sig. fig.) or
cos POD 
OP
131 (cor. to 3 sig. fig.) (rejected)
5

10
POD  60
(c) By similar argument, ROC = 60.
POD  POR  ROC  180 (adj. 
60  POR  60  180
POR  60
st. line)

Area of the shaded region


= area of sector OPR  area of △OPR

60 1
    102    10  10  sin 60  cm 2
 360 2 
 9.06 cm (cor. to 3 sig. fig.)
2

Exercise 9B (p. 178)


Level 1
1. By the sine formula,
BC AC

sin A sin B
x cm 4 cm

sin 75 sin 44
4 sin 75
x 
sin 44
 5.56 (cor. to 3 sig. fig.)

2. By the sine formula,

17
9 Basic Properties of CirclesMore about Trigonometry (II)

4. By the sine formula, A  B  C  180


9. 57  B  81  180 ( sum of △)
AC AB B  42

sin B sin C
9 cm 10 cm

sin x sin 48
9 sin 48
sin x 
10
x  42.0 (cor. to 3 sig. fig.) or
138 (cor. to 3 sig. fig.) (rejected)

5. By the sine formula,


a b

sin A sin B
6 cm 12 cm

sin 60 sin B
12 sin 60
sin B 
6
 1.73 (cor. to 3 sig. fig.)
∵ sin B  1 for all values of B.
∴ There are no solutions for B.

6. By the sine formula,


a b

sin A sin B
3 cm 2 cm

sin 60 sin B
2 sin 60
sin B 
3
1
B  90

7. By the sine formula,

a b

sin A sin B
12 cm 6 cm

sin 60 sin B
6 sin 60
sin B 
12
B  25.7 (cor. to 3 sig. fig.) or
154 (cor. to 3 sig. fig.) (rejected)

8. By the sine formula,


a b

sin A sin B
6 cm 6 cm

sin 70 sin B
6 sin 70
sin B 
6
B  70 or 110  (rejected)

18
Certificate Mathematics in Action Full Solutions 4B

By the sine formula, c b


a b 
 sin C sin B
sin A sin B c 10 cm
a 8 cm 
 sin 23 sin 32
sin 57 sin 42 10 sin 23
8 sin 57 c  cm
a  cm sin 32
sin 42  7.37 cm (cor. to 3 sig. fig.)
 10.0 cm (cor. to 3 sig. fig.)
By the sine formula, 12. By the sine formula,
c b
 c b
sin C sin B 
c 8 cm sin C sin B
 8 cm 5.5 cm
sin 81 sin 42 
8 sin 81 sin C sin 36
c  cm 8 sin 36
sin 42 sin C 
 11 .8 cm (cor. to 3 sig. fig.) 5.5
C  58.8 (cor. to 3 sig. fig.) or 121 (cor. to 3 sig. fi
A  B  C  180 When C = 58.8,
( sum of △)
A  B  C  180
10. A  110   26  180 ( sum of △)
A  44

A  36  58.755  180


By the sine formula,
b a

sin B sin A
b
sin 110 

6 cm
sin 44
A  85.2 (cor. to 3 sig. fig.)
6 sin 110 
b  cm By the sine formula,
sin 44 a b
 8.12 cm (cor. to 3 sig. fig.) 
sin A sin B
By the sine formula, a 5.5 cm

c a sin 85.245 sin 36

sin C sin A 5.5 sin 85.245
a  cm
c 6 cm sin 36

sin 26 sin 44  9.32 cm (cor. to 3 sig. fig.)
6 sin 26
c  cm When C = 121,
sin 44 A  B  C  180 ( sum of △)
 3.79 cm (cor. to 3 sig. fig.) A  36  121.245  180
A  22.8 (cor. to 3 sig. fig.)
A  B  C  180 By the sine formula,
11. 125  32  C  180 ( sum of △)
a b
C  23

sin A sin B
By the sine formula, a 5.5 cm

a b sin 22.755 sin 36

sin A sin B 5.5 sin 22.755
a 10 cm a  cm
 sin 36
sin 125 sin 32  3.62 cm (cor. to 3 sig. fig.)
10 sin 125
a  cm
sin 32 13. By the sine formula,
 15.5 cm (cor. to 3 sig. fig.)
By the sine formula,

19
9 Basic Properties of CirclesMore about Trigonometry (II)

b c AC BC
 
sin B sin C sin ABC sin BAC
b c x
 cm 40 cm
sin 45 sin 30 sin 50 
c sin 45 sin 30 sin 20
b  40 sin 30 sin 50
sin 30 x 
sin 20
b  2c
Take b =
 44.8 (cor. to 3 sig. fig.)
2 ,
2  2c
ABD  BAD  ADB  180
c 1 16. ( sum of △)
∴ The possible values of b and c are 2 and 1 ABD  100  30  180
respectively. (or any other reasonable answers) ABD  50
DBC  ADB (alt. s, BC // AD)
14. Take B = 90, and by the sine formula,
 30
b a DBC  BDC  BCD  180
 ( sum
sin B sin A 30  80  BCD  180
b 6 cm BCD  70

sin 90 sin 30 of △)
6 sin 90
b cm By the sine formula,
sin 30 BD AD
 12 cm 
sin BAD sin ABD
A  B  C  180 ( sum of △)
BD 18 cm
30  90  C  180 
sin 100 sin 50
C  60
18 sin 100
By the sine formula, BD  cm
c a sin 50
 By the sine formula,
sin C sin A
BC BD
c 6 cm 
 sin BDC sin BCD
sin 60 sin 30
18 sin 100
6 sin 60 x cm cm
c 
sin 30
cm  sin 50
sin 80 sin 70
 6 3 cm 18 sin 100 sin 80
∴ The possible values for b, c, B and C are 12 cm, x 
sin 50 sin 70
6 3 cm , 90 and 60 respectively. (or any
 24.3 (cor. to 3 sig. fig.)
other reasonable answers)

Level 2 17. In △ABM,


ABC  BAC  ACD BAM  AMB  ABM  180
15. (ext.  of △) ( sum
30  BAC  50 30  AMB  45  180
BAC  20 AMB  105
Consider △ACD. of △)
AD
sin 50  By the sine formula,
AC
AB BM
x 
AC  cm sin AMB sin BAM
sin 50 x cm 6 cm
By the sine formula, 
sin 105 sin 30
6 sin 105
x 
sin 30
 11 .6 (cor. to 3 sig. fig.)

20
Certificate Mathematics in Action Full Solutions 4B

AM BM ABC  BAC  ACB  180 ( sum



sin ABM sin BAM 40  25  ACB  180
y cm 6 cm ACB  115 
 of △)
sin 45 sin 30
6 sin 45
y 
sin 30
 8.49 (cor. to 3 sig. fig.)
AMC  BAM  ABM (ext.  of △)
 30  45
 75
In △AMC,
MAC  AMC  ACM  180 (
30  75  ACM  180
ACM  75
sum of △)

∵ ACM  AMC
AC  AM

(sides opp.
equal s)
z  y

 8.49 (cor. to 3 sig. fig.)

18. In △ACD,
AC  AD

ACD  ADC
 65
(given)
(base s, isos. )

CAD  ACD  ADC  180


CAD  65  65  180
CAD  50
( sum of )

By the sine formula,


CD AC

sin CAD sin ADC
x cm 7 cm

sin 50 sin 65
7 sin 50
x 
sin 65
 5.92 (cor. to 3 sig. fig.)
In △ABC,
ABC  BAC  ACD (ext.  of
40  BAC  65
BAC  25
△)

21
9 Basic Properties of CirclesMore about Trigonometry (II)

By the sine formula, b a


BC AC 
 sin B sin A
sin BAC sin ABC 12 cm 9 cm
y cm 7 cm 
 sin B sin 70
sin 25 sin 40
12 sin 70
7 sin 25 sin B 
y  9
sin 40
 1.25 (cor. to 3 sig. fig.)
 4.60 (cor. to 3 sig. fig.)
∵ sin B  1 for all values of B.
AB AC ∴ There are no solutions for B.

sin ACB sin ABC
z cm 7 cm

sin 115  sin 40 (cor. to 3 sig. fig.)
Q sin B ?1 for all values of B.
7 sin 115 
z  \ There are no solutions for B.
sin 40
 9.87 (cor. to 3 sig. fig.)

19. By the sine formula,

b a

sin B sin A
12 cm 8 cm

sin B sin 150
12 sin 150
sin B 
8
3

4
B  48.6 (cor. to 3 sig. fig.) (rejected) or
131 (cor. to 3 sig. fig.) (rejected)
∴ There are no solutions for B.

20. By the sine formula,


b a

sin B sin A
6 cm 3.5 cm

sin B sin 30
6 sin 30
sin B 
3.5
6

7

B  59.0 (cor. to 3 sig. fig.) or


121 (cor. to 3 sig. fig.)

21. By the sine formula,

22
Certificate Mathematics in Action Full Solutions 4B

22. By the sine formula, b c


a b 
 sin B sin C
sin A sin B 6.5 cm 9 cm
12 cm 15 cm 
 sin B sin 55
sin 45 sin B 6.5 sin 55
15 sin 45 sin B 
sin B  9
12 ∴
5 2
 B  36.3 (cor. to 3 sig. fig.) or
8
∴ 144 (cor. to 3 sig. fig.) (rejected)
A  B  C  180 ( sum of △)
B  62.1 (cor. to 3 sig. fig.) or A  36.271  55  180
118  (cor. to 3 sig. fig.) A  88.7 (cor. to 3 sig. fig.)

23. By the sine formula,

b c \ (cor. to 3 sig. fig.) or


 144?(cor. to 3 sig. fig.) (rejected)
sin B sin C
(?sum of △)
6 cm 7 cm

sin B sin 50
6 sin 50 (cor. to 3 sig. fig.)
sin B  By the sine formula,
7
a c
B  41.0 (cor. to 3 sig. fig.) or 
sin A sin C
139 (cor. to 3 sig. fig.) (rejected) a 9 cm

A  B  C  180 ( sum of △) sin 88.729 sin 55
A  41.042  50  180 9 sin 88.729
a  cm
A  89.0 (cor. to 3 sig. fig.) sin 55
By the sine formula,  11.0 cm (cor. to 3 sig. fig.)
a c

sin A sin C
a 7 cm
 (cor. to 3 sig. fig.)
sin 88.958 sin 50 26. By the sine formula,
7 sin 88.958 c a
a  cm 
sin 50 sin C sin A
 9.14 cm (cor. to 3 sig. fig.) 10 cm 7.5 cm

sin C sin 43
24. By the sine formula, 10 sin 43
sin C 
a b 7.5
 C  65.4 (cor. to 3 sig. fig.) or
sin A sin B
14 cm 8 cm 115 (cor. to 3 sig. fig.)

sin A sin 62
14 sin 62
sin A 
8
\ (cor. to 3 sig. fig.) or
 1.55 (cor. to 3 sig. fig.) (cor. to 3 sig. fig.)
∵ sin A  1 for all values of A. When C = 65.4,
∴ There are no solutions for A. A  B  C  180 ( sum of △)
∴ No triangle can be formed. 43  B  65.413  180 ?
B  71.6 (cor. to 3 sig. fig.)
(cor. to 3 sig. fig.) (?sum of △)
sin A ?1 for all values of A.
\ There are no solutions for A. (cor. to 3 sig. fig.)
\ No triangle can be formed. By the sine formula,
25. By the sine formula,

23
9 Basic Properties of CirclesMore about Trigonometry (II)

b a CBA  CBT  180



sin B sin A 48  CBT  180
b 7.5 cm CBT  132

sin 71.587 sin 43
7.5 sin 71.587
b  cm (adj. s 苫 on
sin 43
 10.4 cm (cor. to 3 sig. fig.) st. line)

When C = 115,
A  B  C  180 ( sum of △)
43  B  114.587  180 By the sine formula,
B  22.4 (cor. to 3 sig. fig.) CT BC

By the sine formula, sin CBT sin CTB
b a 10 sin 25
 CT cm
sin B sin A  sin 107
b 7.5 cm sin 132 sin 23
 10 sin 25 sin 132
sin 22.413 sin 43 CT  cm
7.5 sin 22.413 sin 23 sin 107
b  cm  8.41 cm (cor. to 3 sig. fig.)
sin 43
 4.19 cm (cor. to 3 sig. fig.)

(cor. to 3 sig. fig.)


(cor. to 3 sig. fig.)
CAB  ACB  CBA (180
sumof △) 28. ∵ ABGF and CDEG are two squares.
27. BGF = 90, CGE = 90, BG = FG and
25  ACB  48  180 (?

ACB  107 CG = EG = 2 cm
sum of △) Consider △BCG.
CBG  BGC  BCG  180 (
CBG  105  30  180
By the sine formula, CBG  45
sum of △)
BC AB

sin CAB sin ACB By the sine formula,
BC 10 cm BG CG
 
sin 25 sin 107 sin BCG sin CBG
10 sin 25 BG 2 cm
BC  cm 
sin 107 sin 30 sin 45
BCT  CAB ( 2 sin 30
 25 BG  cm
sin 45
(s at
 1 cm a pt.)

BGC  BGF  FGE  CGE  360


(?in alt. segment) 105  90  FGE  90  360
BCT  CTB  CBA FGE  75
Area of the hexagon ABCDEF
25  CTB  48 = area of △BCG + area of ABGF + area of △EFG +
CTB  23 area of CDEG

(ext.  ?of △)

24
Certificate Mathematics in Action Full Solutions 4B

1
  BG  CG  sin BGC  BG 2 
2
1
 FG  EG  sin FGE  CG 2
2
 1
 1  2  sin 105  12  
 2 
  1  cm
2

  1  2  sin 75   ( 2 )2 
 2 
 4.37 cm 2 (cor. to 3 sig. fig.)

Exercise 9C (p. 185)


Level 1
1. By the cosine formula,
b 2  a 2  c 2  2ac cos B
b 10 2  8 2  2(10)(8) cos 40 cm

x 10 2  8 2  2(10)(8) cos 40


 6.44 (cor. to 3 sig. fig.)

25
9 Basic Properties of CirclesMore about Trigonometry (II)

2. By the cosine formula, a 2  b 2  c 2  2bc cos A


a 2  b 2  c 2  2bc cos A
a 17 2  12 2  2(17)(12) cos 55 cm
a 14  9  2(14)(9) cos 145 cm
2 2
 14.1 cm (cor. to 3 sig. fig.)
x 14 2  9 2  2(14)(9) cos 145
a2  c2  b2
 22.0 (cor. to 3 sig. fig.) cos B 
2ac
14.106 2  12 2  17 2
3. By the cosine formula,  
b2  c2  a2 2(14.106)(12)
cos A  B  80.8 (cor. to 3 sig. fig.)
2bc
4.5  7 2  6 2
2 A  B  C  180 ( sum of △)
cos  55  80.825  C  180
2(4.5)(7)
C  44.2 (cor. to 3 sig. fig.)
  58.1 (cor. to 3 sig. fig.)

4. By the cosine formula, 8. By the cosine formula,


a2  c2  b2 b 2  a 2  c 2  2ac cos B
cos B 
2ac
b 11 2  8 2  2(11)(8) cos 21 cm
9  6 2  112
2
cos   4.55 cm (cor. to 3 sig. fig.)
2(9)(6)
b2  c2  a2
1 cos A 
 2bc
27
  92.1 (cor. to 3 sig. fig.) 4.5486 2  8 2  112

2(4.5486)(8)
5. The largest angle of △ABC is C. ∴  sum
A  120 (cor. (to of △
3 sig. )
fig.)
By the cosine formula, A  B  C  180
a2  b2  c2 119.93  21  C  180
cos C 
2ab C  39.1 (cor. to 3 sig. fig.)
8  8.5 2  9 2
2
 9. By the cosine formula,
2(8)(8.5)
b2  c2  a2
13 cos A 
 2bc
32
C  66.0 (cor. to 3 sig. fig.) 12 2  10 2  15 2

∴ The largest angle of △ABC is 66.0. 2(12)(10)
19
BAD  ABC  180 
6. (int. s, AD //
240
BAD  60  180 A  85.5 (cor. to 3 sig. fig.)
BAD  120
BC) a2  c2  b2
cos B 
2ac
AD = BC = 12 cm (opp. sides of // gram)
Consider △ABC. 15  10 2  12 2
2

2(15)(10)
AC 2  AB 2  BC 2  2( AB )( BC ) cos ABC 181

AC  8 2  12 2  2(8)(12) cos 60 cm 300
 10.6 cm (cor. to 3 sig. fig.) B  52.9 (cor. to 3 sig. fig.)
Consider △ABD. A  B  C  180 ( sum of △)
85.4593  52.8910  C  180
BD 2  AB 2  AD 2  2( AB )( AD ) cos BAD
C  41.6 (cor. to 3 sig. fig.)
BD  8 2  12 2  2(8)(12) cos 120 cm
 17.4 cm (cor. to 3 sig. fig.) 10. By the cosine formula,

7. By the cosine formula,

26
Certificate Mathematics in Action Full Solutions 4B

b2  c2  a2 AC 2  BC 2  AB 2
cos A  cos ACB 
2bc 2( AC )( BC )
10  5 2  7.5 2
2
4 2  6 2  52
 
2(10)(5) 2( 4)(6)
11 9
 
16 16
A  46.6 (cor. to 3 sig. fig.) ECD  ACB
a 2  c2  b2 cos ECD  cos ACB (vert. opp. s)
cos B 
2ac 9

7.52  52  102 16
 By the cosine formula,
2(7.5)(5)
1 ED 2  CE 2  CD 2  2(CE )(CD ) cos ECD

4
 9 
B  104 (cor. to 3 sig. fig.) ED  7 2  8 2  2(7)(8)  cm
 16 
A  B  C  180 ( sum of △)
x  7.07 (cor. to 3 sig. fig.)
46.567  104.478  C  180
14.
C  29.0 (cor. to 3 sig. fig.)

Level 2
11. By the cosine formula,

AC 2  AB 2  BC 2  2( AB )( BC ) cos ABC
AC  4 2  6 2  2( 4)(6) cos 100 cm
x  7.77 (cor. to 3 sig. fig.) Join AC.
By the cosine formula, By the cosine formula,

AD 2  AC 2  DC 2  2( AC )( DC ) cos ACD AC 2  BC 2  AB 2  2( BC )( AB ) cos ABC


AD  7.7676 2  7 2  2(7.7676)(7) cos 42 cm AC  12 2  13 2  2(12)(13) cos 75 cm
y  5.34 (cor. to 3 sig. fig.)  15.240 cm
By the cosine formula,
12. By the cosine formula, CD 2  AD 2  AC 2
cos ADC 
2(CD )( AD)
AD 2  AB 2  BD 2  2( AB )( BD) cos ABD
11 2  10 2  15.240 2
AD  32  6 2  2(3)(6) cos 48 cm cos  
2(11)(10)
 4.57 cm (cor. to 3 sig. fig.)
  92.9 (cor. to 3 sig. fig.)
∵ ABCD is a parallelogram.
∴ BC = AD and DC = AB = 3 cm
BC  AD 15. Consider △ABD.
By the cosine formula,
x  4.57 (cor. to 3 sig. fig.)
BD 2  AD 2  AB 2
By the cosine formula, cos ADB 
BC 2  BD 2  DC 2 2( BD)( AD )
cos CBD 
2( BC )( BD) ( 4  5) 2  82  62

4.5729 2  6 2  32 2(4  5)(8)
cos  
2( 4.5729)(6) 109

  29.2 (cor. to 3 sig. fig.) 144
Consider △ACD.
By the cosine formula,
13. By the cosine formula,

27
9 Basic Properties of CirclesMore about Trigonometry (II)

AC 2  CD 2  AD 2  2(CD )( AD) cos ADC BCD  ABC  180 (int. s,


BCD  70  180
 109 
AC  5 2  8 2  2(5)(8)  cm BCD  110 
 144  AB // DC)
x  5.33 (cor. to 3 sig. fig.)
Consider △BCD.
16. Consider △ABD.
By the cosine formula, BD 2  DC 2  BC 2  2( DC )( BC ) cos BCD
BD  11 2  8 2  2(11)(8) cos 110  cm
AD 2  BD 2  AB 2  2( BD )( AB ) cos ABD
 15.7 cm (cor. to 3 sig. fig.)
AD  (7  5) 2  10 2  2(7  5)(10) cos 36 cm (b) ∵ ABCD is a parallelogram.
x  7.06 (cor. to 3 sig. fig.) BK  KD
Consider △ABC. 1
By the cosine formula, BD
∴ 2
AC 2  BC 2  AB 2  2( BC )( AB ) cos ABC 1 
   15.659  cm
AC  7 2  10 2  2(7)(10) cos 36 cm 2 
 5.9781 cm  7.830 cm
Consider △ACD. CK  KA
By the cosine formula, 1
AD 2  AC 2  CD 2  AC
cos CAD  2
2( AD)( AC ) 1 
   11 .172  cm
7.0595 2  5.97812  5 2 2 
cos  
2(7.0595)(5.9781)  5.586 cm
  44.1 (cor. to 3 sig. fig.) Consider △BCK.
By the cosine formula,
17. (a) Consider △ABC.
By the cosine formula, CK 2  BK 2  BC 2
cos BKC 
BC  AC  AB
2 2 2
2(CK )( BK )
cos C 
2( BC )( AC ) 5.586 2  7.830 2  8 2

(5  6) 2  (7  3) 2  9 2 2(5.586)(7.830)

2(5  6)(7  3) BKC  71.0 (cor. to 3 sig. fig.)
7 ∴ The acute angle between the diagonals is
 71.0.
11 19. ∵ b : c = 3 : 2
7 b 3
∴ cos C  
11 ∴ c 2
(b) Consider △CMN.
3
By the cosine formula, b c
2
MN 2  NC 2  MC 2  2( NC )(MC ) cos C By the cosine formula,

 7 
MN  6 2  3 2  2(6)(3)  cm
 11 
 4.70 cm (cor. to 3 sig. fig.)

18. (a) Consider △ABC.


By the cosine formula,

AC 2  BC 2  AB 2  2( BC )( AB ) cos ABC
AC  8 2  112  2(8)(11) cos 70 cm
 11.2 cm (cor. to 3 sig. fig.)
∵ ABCD is a parallelogram.
∴ DC = AB = 11 cm

28
Certificate Mathematics in Action Full Solutions 4B

a 2  b 2  c 2  2bc cos A
2
3  3 
14 2 cm 2   c   c 2  2 c (c ) cos 60
2  2 
9 3
196 cm 2  c 2  c 2  c 2
4 2
7 2
196 cm  c
2
4
784
c cm
7
28
 cm
7
 4 7 cm
3
b c
2
3 
   4 7  cm
2 
 6 7 cm

20. (a) Consider △ABC.


By the cosine formula,

AC 2  BC 2  AB 2  2( BC )( AB ) cos ABC
 [32  4 2  2(3)(4) cos  ] cm 2
 ( 25  24 cos  ) cm 2
Consider △ADC.
ABC  ADC  180 (opp. s, cyclic
  ADC  180
ADC  180  
quad.)

By the cosine formula,

AC 2  DC 2  AD 2  2( DC )( AD) cos ADC


 [6 2  5 2  2(6)(5) cos (180   )] cm 2
 (61  60 cos  ) cm 2
25  24 cos   61  60 cos 
∴  36  84 cos 
3
cos   
7
 3
AC  25  24   cm
 7
(b)
247
 cm
7
 5.94 cm (cor. to 3 sig. fig.)

29
9 Basic Properties of CirclesMore about Trigonometry (II)

Exercise 9D (p. 189) s – c = (14 – 7) cm = 7 cm


Level 1 ∴ Area of △ABC
1. a = b = c = 6 cm
abc  s ( s  a )( s  b)( s  c )
Let s  .
2  14  5  2  7 cm 2
666  31.3 cm 2 (cor. to 3 sig. fig.)

s cm
2
 9 cm abc
5. Let s  , where a = 25 m, b = 23 m and
s – a = (9 – 6) cm = 3 cm 2
s – b = (9 – 6) cm = 3 cm c = 18 m.
s – c = (9 – 6) cm = 3 cm 25  23  18
∴ Area of △ABC ∴
s m
2
 s ( s  a )( s  b)( s  c )  33 m
s – a = (33 – 25) m = 8 m
 9  3  3  3 cm 2 s – b = (33 – 23) m = 10 m
 15.6 cm 2 (cor. to 3 sig. fig.) s – c = (33 – 18) m = 15 m
 s ( s  a )( s  b)( s  c )
2. a = 30 cm, b = c = 19 cm Area of △ABC
 33  8  10  15 m 2
abc
Let s  .  60 11 m 2
2
1
30  19  19 Area of △ABC   BC  AD
∴ s cm 2
2
1
 34 cm 60 11 m 2   25 m  h
s – a = (34 – 30) cm = 4 cm 2
s – b = (34 – 19) cm = 15 cm 60 11  2
s – c = (34 – 19) cm = 15 cm h m
∴ Area of △ABC
25
 15.9 m (cor. to 3 sig. fig.)
 s ( s  a )( s  b )( s  c )
 34  4  15  15 cm 2 abc
6. Let s  , where a = 10 cm, b = 14 cm and
 175 cm 2
(cor. to 3 sig. fig.) 2
c = 20 cm.

abc
10  14  20
∴ s
3. Let s  , where a = 14 cm, b = 15 cm and
cm
2 2
c = 13 cm.  22 cm
14  15  13 s – a = (22 – 10) cm = 12 cm

s cm s – b = (22 – 14) cm = 8 cm
2 s – c = (22 – 20) cm = 2 cm
 21 cm  s ( s  a )( s  b)( s  c )
s – a = (21 – 14) cm = 7 cm Area of △ABC
s – b = (21 – 15) cm = 6 cm  22  12  8  2 cm 2
s – c = (21 – 13) cm = 8 cm  8 66 cm 2
∴ Area of △ABC
1
Area of △ABC   BC  AD
 s ( s  a )( s  b)( s  c ) 2
1
 21  7  6  8 cm 2 8 66 cm 2  10 cm  h
2
 84 cm 2
8 66  2
h cm
abc 10
4. Let s  , where a = 9 cm, b = 12 cm and  13.0 cm (cor. to 3 sig. fig.)
2
c = 7 cm.
9  12  7 Level 2
∴ s cm 7.
2
 14 cm
s – a = (14 – 9) cm = 5 cm
s – b = (14 – 12) cm = 2 cm

30
Certificate Mathematics in Action Full Solutions 4B

 s1 ( s1  AB )( s1  OB)( s1  OA) 
s 2 ( s 2  BC )( s 2  OC )( s 2  OB)
Join PR.
PR 2  RS 2  SP 2  ( 13  3  5  5  10.5  5.5  2.5  2.5 ) cm 2
(Pyth. theorem)
 50.2 cm 2 (cor. to 3 sig. fig.)
PR  6 2  8 2 cm
 10 cm 9.
Consider △PQR.
pqr
Let s  , where p = 6 cm, q = 10 cm and
2
r = 7 cm.
5  10  7 Draw
s cm
∴ 2 AE 
BC and DF  BC.
 11 cm
∵ AD // BC
s – p = (11 – 5) cm = 6 cm ∴ AE = DF
s – q = (11 – 10) cm = 1 cm
s – r = (11 – 7) cm = 4 cm 1
  BC  AE
Area of quadrilateral PQRS Area of △ABC 2
 area of △PQR + area of △PRS 1
1   BC  DF
 s ( s  p )(s  q )( s  r ) 
 RS  PS 2
2  area of △BCD
 1  Consider △ABC.
  11  6  1 4   6  8  cm 2 abc
 2  Let s  , where a = 12 cm, b = 10 cm and
 40.2 cm (cor. to 3 sig. fig.)
2 2
c = 6 cm.
12  10  6
8.

s cm
2
 14 cm
s – a = (14 – 12) cm = 2 cm
s – b = (14 – 10) cm = 4 cm
s – c = (14 – 6) cm = 8 cm
Area of △BCD  area of △ABC
Join OB.
Consider △OBA.
OB = OA = 8 cm  s ( s  a )( s  b )( s  c )
AB  OB  OA  14  2  4  8 cm 2
Let s1  .
2  29.9 cm 2 (cor. to 3 sig. fig.)
10  8  8
s 
∴ 1
cm 10. Consider △ACD.
2
DAC  ADC  ACD  180 (
 13 cm
s1 – AB = (13 – 10) cm = 3 cm 37  ADC  80  180
s1 – OB = (13 – 8) cm = 5 cm ADC  63
s1 – OA = (13 – 8) cm = 5 cm sum of △)
Consider △OCB.
OC = OB = 8 cm By the sine formula,
BC  OC  OB AC CD
Let s 2  . 
2 sin ADC sin DAC
588 8 sin 63
∴ 2
s  cm AC  cm
2 sin 37
 10.5 cm  11 .844 cm
s2 – BC = (10.5 – 5) cm = 5.5 cm
s2 – OC = (10.5 – 8) cm = 2.5 cm BC  AC  AB
Let s  .
s2 – OB = (10.5 – 8) cm = 2.5 cm 2
Area of quadrilateral OCBA 15  11 .844  10
 area of △OBA + area of △OCB ∴ s 2
cm

 18.422 cm
s – BC = (18.422 – 15) cm = 3.422 cm

31
9 Basic Properties of CirclesMore about Trigonometry (II)

s – AC = (18.422 – 11.844) cm = 6.578 cm


s – AB = (18.422 – 10) cm = 8.422 cm
Area of △ABC

 s ( s  BC )( s  AC )( s  AB )
 18.422  3.422  6.578  8.422 cm 2
 59.1 cm 2 (cor. to 3 sig. fig.)

11.

32
Certificate Mathematics in Action Full Solutions 4B

Join BE and BD. With the notations in the figure, consider △APH.
Consider △ABE. PAH  180  130
By the cosine formula,
 50
By the cosine formula,
BE 2
 AE  AB  2( AE )( AB ) cos EAB
2 2

BE  2 2  4 2  2( 2)( 4) cos 110  km HP 2  AH 2  AP 2  2( AH )( AP ) cos PAH


 5.047 km HP  400 2  300 2  2( 400)(300) cos 50 m
Consider △BCD.
By the cosine formula,  309 m (cor. to 3 sig. fig.)
∴ Harry is 309 m away from the park.
BD 2  BC 2  CD 2  2( BC )(CD) cos BCD
3.
BD  3.5 2  2.8 2  2(3.5)( 2.8) cos 130 km
 5.717 km
Consider △BDE.
DE  BE  BD
Let s  .
2
2  5.047  5.717

s km
2
 6.382 km
s – DE = (6.382 – 2) km = 4.382 km
s – BE = (6.382 – 5.047) km = 1.335 km
s – BD = (6.382 – 5.717) km = 0.665 km
Area of the lake With the notations in the figure,
 area of △ABE + area of △BDE + area of △BCD BCD  180  150
1
 30
  AB  EA  sin EAB Consider △ABC.
2
 s ( s  DE )( s  BE )( s  BD ) BC 2  CA 2  AB 2
cos ACB 
1
 BC  CD  sin BCD 2( BC )(CA)
2
 1  48 2  60 2  55 2
  4  2  sin 110    
 2  2(48)(60)
  6.382  4.382  1.335  0.665   km 2
 ACB
  60.011 
 1 
 2  3.5  2.8  sin 130 
  CA 2  AB 2  BC 2
cos CAB 
 12.5 km 2 (cor. to 3 sig. fig.) 2(CA)( AB )
60 2  55 2  48 2
Exercise 9E (p. 196) 
2(60)(55)
Level 1
CAB  49.103
1. By the cosine formula, ACD  ACB  BCD
AP  BP  AB
2 2 2
 60.011   30
cos APB 
2( AP)( BP )  30.011 
12 2  15 2  7.32 2 FAC  ACD (alt. s, FA // CD)
  30.011 
2(12)(15)
FAB  FAC  CAB
APB  28.8 (cor. to 3 sig. fig.)
 30.011   49.103
2.  79.1 (cor. to 3 sig. fig.)
∴ The true bearing of B from A is 079.1.

DAB  30  90


4.  120
ABD  90  48
 42
In △ABD,

33
9 Basic Properties of CirclesMore about Trigonometry (II)

ADB  180  DAB  ABD (


 180  120  42
 18
sum of △)

34
Certificate Mathematics in Action Full Solutions 4B

By the sine formula, AD BD


DB AB 
 sin ABD sin BAD
sin DAB sin ADB AD 200 m
DB 50 m 
 sin 35 sin 95
sin 120 sin 18 200 sin 35
50 sin 120 AD  m
DB  m sin 95
sin 18
In △BDC,
DC
sin DBC 
DB
DC  DB sin DBC
 50 sin 120 
  sin 48  m
 sin 18 
 104 m (cor. to 3 sig. fig.)
∴ The height of the hill is 104 m.

5. In △ABT,
TAB  ATB  TBH (ext.  of )

25  ATB  60
ATB  35
By the sine formula,
TB AB

sin TAB sin ATB
TB 200 m

sin 25 sin 35
200 sin 25
TB  m
sin 35
In △TBH,
TH
sin TBH 
TB
TH  TB sin TBH
 200 sin 25 
  sin 60  m
 sin 35 
 128 m (cor. to 3 sig. fig.)
∴ The height of the tower is 128 m.

6.

Join AC.
In △ABD,
BAD  ABD  ADB  180 ( sum
BAD  35  50  180
BAD  95
of △)

By the sine formula,

35
9 Basic Properties of CirclesMore about Trigonometry (II)

In △ACD, PAB  APB  ABP  180 ( sum


AC 50  60  ABP  180
sin ADC 
AD ABP  70
AC  AD sin ADC of △)
 200 sin 35 
  sin 50  m
 sin 95 
 88.2 m (cor. to 3 sig. fig.)
∴ The height of the building is 88.2 m.

7. In △ABD,
AB
sin ADB 
AD
AB
AD 
sin ADB
40
 m
sin 25
In △ADC,
ADC  90  25  65
ACD  90  35  55
CAD  ADC  ACD  180 ( sum
CAD  65  55  180
CAD  60
of △)

By the sine formula,


CD AD

sin CAD sin ACD
40
m
CD
 sin 25
sin 60 sin 55
 40 
 
CD   sin 25  sin 60  m
 sin 55 
 
 
 100 m (cor. to 3 sig. fig.)

8.

With the notations in the figure,


PAB  CPA (alt. s, CP // AB)
 50
In △APB,
APB  110   50  60

36
Certificate Mathematics in Action Full Solutions 4B

By the sine formula, DC 2  DP 2  CP 2  2( DP)(CP ) cos DPC


PA AB
 DC  80 2  60 2  2(80)(60) cos 99 km
sin ABP sin APB
 107 km (cor. to 3 sig. fig.)
PA 8 km
 ∴ The distance between C and D is 107 km.
sin 70 sin 60
8 sin 70
PA  km
sin 60
 8.68 km (cor. to 3 sig. fig.)
∴ The distance between P and A is 8.68 km.

9.

With the notations


in the figure,
BAL  ALC (alt. s, DA // LC)
 35
BAL  BLA  DBL (ext.  of )

35  BLA  60
BLA  25
ABL  BAL  BLA  180 ( sum
ABL  35  25  180
ABL  120
of △)

By the sine formula,


BA AL

sin BLA sin ABL
BA 50 km

sin 25 sin 120
50 sin 25
BA  km
sin 120
 24.400 km
∴ The average speed of the ship
24.400 km

2h
 12.2 km/h (cor. to 3 sig. fig.)

Level 2

10. (a) DPC  66  33


 99
DP = (40  2) km = 80 km
CP = (30  2) km = 60 km
By the cosine formula,

37
9 Basic Properties of CirclesMore about Trigonometry (II)

(b) BC AC

sin CAB sin CBA
20 km 18.880 km

sin CAB sin 67
20 sin 67
sin CAB 
18.880

With the notations in the figure, CAB  77.192 or 102.808 (rejected)


GDP  DPF (alt. s, GD // PF) CAD  CAB  DAB
 66  77.192  48
Consider △DCP.  29.2 (cor. to 3 sig. fig.)
By the sine formula,
∴ The compass bearing of C from A is N29.2W.
CP DC

sin PDC sin DPC 12. (a)
60 km 107.25 km

sin PDC sin 99
60 sin 99
sin PDC 
107.25

PDC  33.544 or 146.456 (rejected)


With the notations in the figure,
GDP  PDC  CDH  180 (adj. s on st. line)
BCD  180  130 (adj. s on st.
66  33.544  CDH  180
 50
CDH  80.5 (cor. to 3 sig. fig.) line)
∴ The compass bearing of C from D is
S80.5E. By the cosine formula,
BC 2  CA 2  AB 2
11. (a) cos ACB 
2( BC )(CA)
30 2  45 2  65 2

2(30)(45)
ACB  118.782
ACD  ACB  BCD
 118 .782  50
 68.782
FAC  ACD (alt. s, FA // CD)
With the notations in the figure,  68.782
ABF  DAB (alt. s, BF // DA) By the cosine formula,
 48 CA 2  AB 2  BC 2
CBG  CBA  ABF  180 (adj. cos CAB 
2(CA)( AB )
65  CBA  48  180
45 2  65 2  30 2
CBA  67 
2( 45)(65)
s on st. line)
CAB  23.861
Consider △ABC. FAB  FAC  CAB
By the cosine formula,
 68.782  23.861
AC 2  AB 2  BC 2  2( AB )( BC ) cos CBA  92.6 (cor. to 3 sig. fig.)
∴ The true bearing of B from A is 092.6.
AC  12 2  20 2  2(12)(20) cos 67 km
 18.9 km (cor. to 3 sig. fig.) (b) The shortest distance of the ship to island C is
CG, such that CG  AB.
∴ The distance between A and C is 18.9 km.
abc
Let s  , where a = 30 km, b = 45
(b) By the sine formula, 2
km and c = 65 km.

38
Certificate Mathematics in Action Full Solutions 4B

30  45  65 1

s km ∵ Area of △ABC   AB  CG
2 2
 70 km ∴
s – a = (70 – 30) km = 40 km
s – b = (70 – 45) km = 25 km 1
 65 km  CG  70  40  25  5 km 2
s – c = (70 – 65) km = 5 km 2
Area of △ABC
70  40  25  5  2
CG  km
 s ( s  a )( s  b)( s  c ) 65
 70  40  25  5 km 2  18.2 km (cor. to 3 sig. fig.)
∴ The shortest distance of the ship to island C is
18.2 km.

13. (a)

With the notations in the figure,


QAP  AQC (alt. s, PA // QC)
 20
BAQ  BAP  QAP
 55  20
 35
AQB  50  20
 70
ABQ  AQB  BAQ  180
(
ABQ  70  35  180
ABQ  75
sum of △)

Consider △APQ.
PQ
sin QAP 
AQ
10 m
sin 20 
AQ
10
AQ  m
sin 20
By the sine formula,
AB AQ

sin AQB sin ABQ
10
m
AB sin 20

sin 70 sin 75
 10 
 
AB   sin 20   sin 70  m
 sin 75 
 
 
 28.4 m (cor. to 3 sig. fig.)
∴ The length of the string AB is 28.4 m.

39
9 Basic Properties of CirclesMore about Trigonometry (II)

(b) Draw BD  AP.  AD  BD


In △ADB, Length of path II
 ( 2.8  2.7271) km
BD
sin BAD   5.5271 km
AB Difference between the two paths
BD  AB sin BAD  (5.5271  5.3718) km
 28.444 sin 55 m  0.155 km (cor. to 3 sig. fig.)
 23.3 m (cor. to 3 sig. fig.) ∴ The first path is shorter, the difference between
∴ The height of the balloon above the road AP is the two paths is 0.155 km.
23.3 m.
15. HAB  40  15
14.
 25
HBK  55  15
 40
HAB  AHB  HBK (ext.  of )

25  AHB  40
AHB  15
By the sine formula,
HB AB
Join BA.

sin HAB sin AHB
With the notations in the figure,
(alt. s, BA // FC)
HB 10 m
ABC  BCF 
 35
sin 25 sin 15
Consider △ABC. 10 sin 25
HB  m
By the sine formula, sin 15
BC AC Consider △BKH.

sin BAC sin ABC BHK  90  55
BC 2.3 km  35
 BHK  HBK  BKH  180 ( sum
sin 50 sin 35
2.3 sin 50 35  40  BKH  180
BC  km BKH  105
sin 35
of △)
 3.0718 km
ABC  ACB  BAC  180 ( sum of By the sine formula,
35  ACB  50  180 HK HB

ACB  95 sin HBK sin BKH
By the sine formula, 10 sin 25
m
AB AC HK
  sin 15
sin ACB sin ABC sin 40 sin 105
AB 2.3 km  10 sin 25 
  
sin 95 sin 35 HK   sin 15  sin 40  m
2.3 sin 95  sin 105 
AB  km  
sin 35  
Consider △ADB.  10.9 m (cor. to 3 sig. fig.)
By the cosine formula, ∴ The height of the lamppost is 10.9 m.
 
PHQ  HQP  HPA
 2( AB )( AD ) cos BAD
2 2 2
BD AB AD

16. (a) (ext.  of △)


2
2.3 sin 95  

 sin 35


 2.8 2  PHQ  42  65
BD  km
 2.3 sin 95 
2
 sin 35
 ( 2.8) cos 43

PHQ  23
 2.7271 km Consider △HPQ.
 AC  BC By the sine formula,
Length of path I
 ( 2.3  3.0718) km
 5.3718 km

40
Certificate Mathematics in Action Full Solutions 4B

HP PQ

sin HQP sin PHQ
HP 80 m

sin 42 sin 23
80 sin 42
HP  m
sin 23

41
9 Basic Properties of CirclesMore about Trigonometry (II)

In △HAP, With the notations in the figure,


HA HC  HA  CA
sin HPA   HA  KB
HP
HA  HP sin HPA  80 sin 42 
  sin 65  
sin 23 
 m
 80 sin 42   80 sin 35 
  sin 65  m   sin 60 
 sin 23   sin 25  
 30.135 m
 124 m (cor. to 3 sig. fig.)
CK  AB
PKQ  KPQ  KQB
(ext.  of △)  192.187 m
PKQ  35  60 In △HCK,
PKQ  25
Consider △KPQ. HC
By the sine formula, tan HKC 
CK
KQ PQ 30.135 m
 
sin KPQ sin PKQ 192.187 m
KQ 80 m HKC  8.91 (cor. to 3 sig. fig.)

sin 35 sin 25 ∴ The angle of elevation of H from K is
80 sin 35 8.91.
KQ  m
sin 25 17.
In △KBQ,
KB
sin KQB 
KQ
KB  KQ sin KQB
 80 sin 35 
  sin 60  m
 sin 25  
 94.0 m (cor. to 3 sig. fig.)
Join BP.
(b) In △HAP,
AP ABQ  BAP  APQ  BQP  360
cos HPA 
HP ABQ  70  90  (90  38)  360
AP  HP cos HPA ABQ  72
 80 sin 42  Consider △APB.
  cos 65  m By the cosine formula,
 sin 23  
In △KBQ,
BP 2  AP 2  AB 2  2( AP )( AB ) cos BAP
QB
cos KQB 
KQ BP  4 2  5 2  2( 4)(5) cos 70 m
QB  KQ cos KQB  5.2268 m
By the sine formula,
 80 sin 35 
  cos 60  m
 sin 25   AP BP

sin ABP sin BAP
AB  AP  PQ  QB
4m 5.2268 m
 80 sin 42


 cos 65  80   
  sin 23 m sin ABP sin 70
 80 sin 35 
  cos 60  4 sin 70
 sin 25  sin ABP 
 192 m (cor. to 3 sig. fig.) 5.2268
∴ The distance between A and B is 192 m. ABP  45.983 or 134.017 (rejected)
PBQ  ABQ  ABP
(c)
 72  45.983
 26.017
Consider △BPQ.

42
Certificate Mathematics in Action Full Solutions 4B

PQ BP
 AC
sin PBQ sin BQP tan AGC 
PQ 5.2268 m CG
 10
sin 26.017 sin 128 
5.2268 sin 26.017 5
PQ  m AGC  63.4 (cor. to 3 sig. fig.)
sin 128
 2.91 m (cor. to 3 sig. fig.) ∴ The angle between AG and GC is 63.4.

Exercise 9F (p. 209) (b)


Level 1
1. (a)

BG 2  BC 2  CG 2 (Pyth. theorem)
BG  6  6 cm
2 2

 72 cm
The angle between AG
 8.49 cm (cor. to 3 sig. fig.) and plane ADHE is GAH.

BH 2  BG 2  HG 2 (Pyth. theorem) AH 2  AD 2  DH 2 (Pyth. theorem)

BH  ( 72 ) 2  6 2 cm AH  6  5 cm
2 2

 108 cm  61 cm
 10.4 cm (cor. to 3 sig. fig.)
HG
tan GAH 
(b) The angle between BG and plane ABCD is AH
GBC. 8

GC 61
tan GBC 
BC GAH  45.7 (cor. to 3 sig. fig.)
6 ∴ The angle between AG and plane ADHE is

6 45.7.
GBC  45 3. (a)
∴ The angle between BG and plane ABCD is
45.

(c) The angle between BH and plane BCGF is


HBG.

HG
tan HBG  The angle between HB and DB is HBD.
BG
6 BD 2  AB 2  AD 2 (Pyth. theorem)

72
BD  7  5 cm
2 2

HBG  35.3 (cor. to 3 sig. fig.)


 74 cm
∴ The angle between BH and plane BCGF is
35.3.
HD
tan HBD 
2. (a) BD
4

74
HBD  24.9 (cor. to 3 sig. fig.)
∴ The angle between HB and DB is 24.9.

The angle between AG and GC is AGC. (b)

AC 2  AB 2  BC 2 (Pyth. theorem)
AC  8  6 cm
2 2

 10 cm

43
9 Basic Properties of CirclesMore about Trigonometry (II)

The angle between HB and plane CGHD is


BHC.

HC 2  HG 2  GC 2 (Pyth. theorem)
HC  7 2  4 2 cm
 65 cm

BC
tan BHC 
HC
5

65
BHC  31.8 (cor. to 3 sig. fig.)
∴ The angle between HB and plane CGHD is
31.8.

44
Certificate Mathematics in Action Full Solutions 4B

4. (a) The angle between the planes ACFD and BCFE 1


is ACB. QM  QR
Consider △ABC. 2
1 
   8  cm
CA2  BC 2  AB 2 2 
cos ACB 
2(CA)( BC )  4 cm
10 2  122  82 ∵

2(10)(12) AQ 2  AB 2  BQ 2 (Pyth. theorem)
3 AQ  AB  BQ
2 2

4
 AC 2  CR 2
ACB  41.4 (cor. to 3 sig. fig.)
∴ The angle between the planes ACFD and  AR (Pyth. theorem)
BCFE is 41.4. ∴ △AQR is an isosceles triangle.
∴ AM  QR
(b) The angle between the planes ACFD and BCFE ∵ PQ = PR = 5 cm
is ACB. ∴ △PQR is an isosceles triangle.
Consider △ABC. ∴ PM  QR
By the sine formula, ∴ QR is the line of intersection of the planes
AQR and PQR.
AB BC ∴ The angle between the planes AQR and
 PQR is AMP.
sin ACB sin BAC Consider △PQM.
11 cm 9 cm

sin ACB sin 35 PQ 2  PM 2  QM 2 (Pyth. theorem)
11 sin 35 PM  5  4 cm
2 2
sin ACB 
9  3 cm
ACB  44.5 (cor. to 3 sig. fig.) or
135 (cor. to 3 sig. fig.) (rejected) AP
tan AMP 
∴ The angle between the planes ACFD and PM
BCFE is 44.5. 10

5. (a) The angle between the planes ABQP and ACRP
3
is BAC. AMP  73.3 (cor. to 3 sig. fig.)
Consider △ABC. ∴ The angle between the planes AQR and
PQR is 73.3.
AB 2  AC 2  BC 2
cos BAC  6. (a) In △CBE,
2( AB)( AC ) 700 m
5 2  52  82 sin 30 
 BC
2(5)(5) 700
BC  m
7 sin 30

25 Consider △DBC.
BAC  106 (cor. to 3 sig. fig.) BC
cos 55 
∴ The angle between the planes ABQP and BD
ACRP is 106. BC
BD 
(b) cos 55
700
 sin 30 m
cos 55
 2440 m (cor. to 3 sig. fig.)

(b)

Let M be the mid-point of QR.

45
9 Basic Properties of CirclesMore about Trigonometry (II)

8. (a) The angle between VA and plane ABCD is


VAN.
The angle between the path BD and plane ABEF
is DBF.
Consider △DBF. AC  8 2  6 2 cm (Pyth. theorem)
 10 cm
DF 1
sin DBF  ∴ AN  AC  5 cm (property of
BD 2
700 m rectangle)
 Consider △VAN.
2440.826 m
DBF  16.7 (cor. to 3 sig. fig.)
∴ The angle between the path BD and plane
ABEF is 16.7.

7. (a) In △FBE,
60 cm
tan 50 
BE
60
BE  cm
tan 50
In △EBC,
12
EC tan VAN 
sin 30  5
BE
VAN  67.4 (cor. to 3 sig. fig.)
EC  BE sin 30
∴ The angle between VA and plane ABCD is
 60  67.4.
  sin 30  cm
 tan 50 
 25.2 cm (cor. to 3 sig. fig.)

(b)

The
angle between BF and plane ABCD is FBD.
In △FBE,
60 cm
sin 50 
BF
60
BF  cm
sin 50
In △FBD,

FD
sin FBD 
BF
EC

BF
 60 
  sin 30  cm
tan 50 
  
60
cm
sin 50
FBD  18.7 (cor. to 3 sig. fig.)
∴ The angle between BF and plane ABCD is
18.7.

46
Certificate Mathematics in Action Full Solutions 4B

(b)

Let M be the mid-point of AB.


∵ △VAB is an isosceles triangle.
∴ VM  AB
∵ △NAB is an isosceles triangle.
∴ NM  AB
∴ AB is the line of intersection of the planes
VAB and ABCD.
∴ The angle between the planes VAB and
ABCD is VMN.
6
MN  cm
2
 3 cm
Consider △VMN.

12
tan VMN 
3
VMN  76.0 (cor. to 3 sig. fig.)
∴ The angle between the planes VAB and
ABCD is 76.0.

9. (a) The angle between VC and plane ABCD is


VCM.
1
MB  AB
2
1 
   8  cm
2 
 4 cm
In △CMB,

MC 2  MB 2  BC 2 ( Pyth. theorem)
MC  4  6 cm
2 2

 52 cm
In △VCM,

VM
tan VCM 
MC
10 cm

52 cm
VCM  54.2 (cor. to 3 sig. fig.)
∴ The angle between VC and plane ABCD is
54.2.

(b)

47
9 Basic Properties of CirclesMore about Trigonometry (II)

Let N be the mid-point of DC. BD 2  BE 2  DE 2


The angle between the planes VDC and ABCD cos DBE 
is VNM. 2( BD )( BE )
In △VNM,
( 1025 ) 2  ( 1025 ) 2  8 2

VM 2( 1025 ) ( 1025 )
tan VNM 
MN 993

VM 1025

BC DBE  14.4 (cor. to 3 sig. fig.)
10 cm ∴ The angle between BD and plane BCEF is
 14.4.
6 cm
VNM  59.0 (cor. to 3 sig. fig.) 11. (a)
∴ The angle between the planes VDC and
ABCD is 59.0. BD 2  BC 2  CD 2 (Pyth. theorem)

10. (a) The angle between the covers ABCD and BCEF BD  20 2  152 m
is DCE.  25 m
Consider △DCE.
1
BK  BD
CD  CE  DE
2 2 2 2
cos DCE  ∴
25
2(CD )(CE )  m
2
252  252  82
  12.5 m
2( 25)(25) Consider △VBK.
593 VK
 tan VBK 
625 BK
DCE  18.4 (cor. to 3 sig. fig.) VK  BK tan VBK
∴ The angle between the covers ABCD and  12.5 tan 15 m
BCEF is 18.4. Height of the house  VK  DE
(b)  (12.5 tan 15  7) m
 10.3 m (cor. to 3 sig. fig.)

(b)

The angle between BD and plane BCEF is


DBE.

BD 2  AB 2  AD 2 (Pyth. theorem)
BD  252  20 2 cm
 1025 cm
Let M and N be the mid-points of AD and
BE 2  BC 2  CE 2 (Pyth. theorem) HE respectively.
The angle between the planes VAD and ADEH is
BE  20 2  25 2 cm
VMN.
 1025 cm ∵ △VAD is an isosceles triangle.
Consider △BED. ∴ VM  AD
By the cosine formula, ∵ △KAD is an isosceles triangle.
∴ KM  AD
∴ AD is the line of intersection of the planes
VAD and ABCD.
∴ The angle between planes VAD and
ABCD is VMK.

48
Certificate Mathematics in Action Full Solutions 4B

1
KM  FE
2
1 
   15  cm
 2 
 7.5 cm
Consider △VMK.
VK
tan VMK 
KM
12.5 tan 15

7.5
VMK  24.065
VMN  VMK  KMN
 24.065  90
 114  (cor. to 3 sig. fig.)
∴ The angle between the planes VAD and
ADEH is 114.

49
9 Basic Properties of CirclesMore about Trigonometry (II)

12. (a)

Consider △EFG.

EG 2  EF 2  FG 2 (Pyth. theorem)
EG  10  18 cm
2 2

 424 cm
Consider △AEG.

AG 2  AE 2  EG 2 (Pyth. theorem)
AG  14 2  424 cm
 620 cm
Consider ABGH.
∵ BA = GH, AH = BG and ABG = 90
∴ ABGH is a rectangle.
∴ AK = KG = BK = KH (property of rectangle)
KG  KH
1
∴  AG
2
620
 cm
2
Consider △KGH.

KG 2  KH 2  GH 2
cos GKH 
2( KG )( KH )
2 2
 620   620 
     10 2
 2   2 
   

 620  620 
2 
 2  2 
 
21

31
GKH  47.4 (cor. to 3 sig. fig.)

(b) ∵ The angle between the planes KGH and


EFGH = the angle between the planes
ABGH and EFGH
∴ The angle between the planes KGH and
EFGH is BGF.
Consider △BFG.

BF
tan BGF 
FG
14

18
BGF  37.9 (cor. to 3 sig. fig.)
∴ The angle between the planes KGH and
EFGH is 37.9.

50
Certificate Mathematics in Action Full Solutions 4B

Level 2
13. (a) Consider △BDC.

BD 2  BC 2  CD 2 (Pyth. theorem)
BD  120  190 cm
2 2

 50 500 cm
 225 cm (cor. to 3 sig. fig.)

(b)

Join CE and
DF.
Distance between D and F
= distance between C and E
Consider △BEC.
By the cosine formula,

CE 2  BC 2  BE 2  2( BC )( BE ) cos CBE
CE  1202  1202  2(120)(120) cos 50 cm
 101 cm (cor. to 3 sig. fig.)
∴ The distance between D and F is 101 cm.

(c)

Join BD and
BF.
The angle between BF and DF is BFD.
BF  BD  50 500 cm
Consider △BFD.
By the cosine formula,

BF 2  DF 2  BD 2
cos BFD 
2( BF )( DF )
( 50 500 ) 2  101.428 2  ( 50 500 ) 2

2( 50 500 ) (101.428)
 77.0 (cor. to 3 sig. fig.)
∴ The angle between BF and DF is 77.0.

14. (a) The angle between the planes AFGD and ABCD
is FAB.

51
9 Basic Properties of CirclesMore about Trigonometry (II)

Consider △FAB.
AB 2  AM 2  BM 2 (Pyth. theorem)
FB
tan FAB  AM  10  5 cm2 2

AB
 75 cm
6
  8.66 cm (cor. to 3 sig. fig.)
9
Consider △DBM.
FAB  33.7 (cor. to 3 sig. fig.)
∴ The angle between the planes AFGD and BD 2  BM 2  MD 2 (Pyth. theorem)
ABCD is 33.7.
MD  10 2  5 2 cm
(b)
 75 cm
 8.66 cm (cor. to 3 sig. fig.)

(b) The angle between AD and plane BCD is


ADM.
Consider △AMD.
By the cosine formula,

The angle between the planes ABCD and EPQH AD 2  MD 2  AM 2


cos ADM 
is EPA. 2( AD )( MD)
∵ P is the mid-point of AB.
10 2  ( 75 ) 2  ( 75 ) 2
9 
∴ AP  cm 2(10)( 75 )
2
 4.5 cm 5

Consider △EAP. 75
ADM  54.7 (cor. to 3 sig. fig.)
EA
tan EPA  ∴ The angle between AD and plane BCD is
AP 54.7.
6
 (c) BC is the line of intersection of the planes ABC
4 .5 and BCD.
EPA  53.1 (cor. to 3 sig. fig.) ∵ AMB = 90 and DMB = 90
∴ The angle between the planes ABCD and ∴ The angle between the planes ABC and
EPQH is 53.1. BCD is AMD.
Consider △AMD.
15. (a) ∵ ABCD is a regular tetrahedron.
∴ AB = AC = AD = BC = BD = CD = 10 cm AM 2  MD 2  AD 2
∵ M is the mid-point of BC. cos AMD 
BM  MC 2( AM )(MD)

1 ( 75 ) 2  ( 75 ) 2  102
 BC 
∴ 2 2( 75 )( 75 )
10 1
 cm 
2 3
 5 cm AMD  70.5 (cor. to 3 sig. fig.)
Consider △ABM and △ACM. ∴ The angle between the planes ABC and
AB  AC (given) BCD is .
BM  CM (given)
16. (a) Consider △HCA.
AM  AM (common side)
HA
∴ △ABM  △ACM (SSS) tan HCA 
CA
AMB  AMC
(corr. s,  20 cm
180 tan 30 
 CA
2 20
 90 CA  cm
tan 30
△s)
 20 3 cm
By similar argument, BMD = CMD = 90 Consider △HBA.
Consider △ABM.

52
Certificate Mathematics in Action Full Solutions 4B

HA
tan HBA 
BA
20 cm
tan 60 
BA
20
BA  cm
tan 60
20
 cm
3

53
9 Basic Properties of CirclesMore about Trigonometry (II)

Consider △ACB. The angle between the planes BDA and ABC is
By the cosine formula, DEC.

BA2  BC 2  CA2
cos ABC 
2( BA)( BC )
2
 20 
 
 3   30  ( 20 3 )
2 2

  
 20 
2 (30)

 3
5

12 3
ABC  104 (cor. to 3 sig. fig.)

(b) ∵ M is the mid-point of BC.


BM  MC
∴ 30
 cm
2
 15 cm
Consider △AMB.
By the cosine formula,

AM 2
 BA 2  BM 2
 2( BA)( BM ) cos ABC
2
 20 
   152 
 3 
AM    cm
 20 
2  (15) cos 103.920
 3 

 21.016 cm
The angle between HM and plane ABC is
HMA.
Consider △HMA.

HA
tan HMA 
AM
20

21.016
HMA  43.6 (cor. to 3 sig. fig.)
∴ The angle between HM and plane ABC is
43.6.

17. (a) The angle between DB and plane ABC is


DBC.
Consider △DBC.

8
sin DBC 
10
DBC  53.1 (cor. to 3 sig. fig.)
∴ The angle between DB and plane ABC is
53.1.

(b)

Draw CE  BA and DE  BA.

54
Certificate Mathematics in Action Full Solutions 4B

Consider △BCD. is ACE.

BD 2  BC 2  CD 2 (Pyth. theorem)
BC  10 2  8 2 cm
 6 cm
By similar argument, AC = 6 cm
Consider △CBE and △CAE.

BC  AC  6 cm (proved)
BEC  AEC  90 (as constructed)
CE  CE (common side)
∴ △CBE  △CAE (RHS)
BE = AE (corr. sides,  △s)
Consider △ACB.

AB 2  AC 2  CB 2 (Pyth. theorem)
AB  6  6 cm
2 2

 72 cm
1
BE  AB
2
72
 cm
2
Consider △BEC.

BC 2  BE 2  CE 2 (Pyth. theorem)
2
 72 
CE  62    cm
 2 
 
 18 cm
Consider △DEC.

DC
tan DEC 
CE
8

18
DEC  62.1 (cor. to 3 sig. fig.)
∴ The angle between the planes BDA and
ABC is 62.1.

18. (a)

Join AE .
BAE = 90
Consider △ABE.

BE 2  AB 2  AE 2 (Pyth. theorem)
AE  102  62 cm
 8 cm
The angle between the planes ABDC and CDFE

55
9 Basic Properties of CirclesMore about Trigonometry (II)

Consider △AEC. and VBN.


∵ AN  VN and BN  VN
AC 2  CE 2  AE 2 ∴ The angle between the planes VAN and
cos ACE  VBN is ANB.
2( AC )(CE )
4 2  82  82

2(4)(8)
1

4
ACE  75.5 (cor. to 3 sig. fig.)
∴ The angle between the planes ABDC and
CDFE is 75.5.

(b)

Let G be the projection of E on plane ABDC.


∴ CGE = 90 and BGE = 90
Consider △CGE.
EG
sin GCE 
CE
EG
sin 75.522 
8 cm
EG  8 sin 75.522 cm
The angle between BE and plane ABDC is
EBG.
Consider △BEG.

EG
sin EBG 
BE
8 sin 75.522

10
EBG  50.8 (cor. to 3 sig. fig.)
∴ The angle between BE and plane ABDC is
50.8.

19. (a) Consider △ABC.

AC 2  AB 2  BC 2 (Pyth. theorem)
AC  4  4 cm
2 2

 32 cm
1
AN  AC (property of square)
2
32
 cm
2
BN  AN (property of square)
32
 cm
2
VN is the line of intersection of the planes VAN

56
Certificate Mathematics in Action Full Solutions 4B

Consider △ABN. Let F be the mid-point of DC.


By the cosine formula,
AN 2  BN 2  AB 2
cos ANB 
2( AN )( BN )
2 2
 32   32 
     42
 2   2 
    
 32  32 
2  
 2  2 
  
0
ANB  90
∴ The angle between the planes VAN and
VBN is 90.

(b)

Let E
be the mid-point of AB.
∵ △VAB is an isosceles triangle.
∴ VE  AB
∵ △NAB is an isosceles triangle.
∴ NE  AB
∴ AB is the line of intersection of the planes
VAB and ABCD.
∴ The angle between the planes VAB and
ABCD is VEN.
1
EN  BC
2
1 
   4  cm
2 
 2 cm
Consider △VEN.

VN
tan VEN 
EN
6

2
VEN  71.6 (cor. to 3 sig. fig.)
∴ The angle between the planes VAB and
ABCD is 71.6.

(c)

57
9 Basic Properties of CirclesMore about Trigonometry (II)

By similar argument as in (b), Consider △EMQ.


VFN = 71.6 (cor. to 3 sig. fig.)
The angle between the planes VAB and VCD is QE 2  EM 2  MQ 2 (Pyth. theorem)
EVF.
2
Consider △VEF.  3  3
2

EVF  VEN  VFN(sum 180of △) EM       cm


 2 2
EVF  71.565  71.565  180
3
EVF(cor.
 36 9fig.)
to 3 .sig.
 cm
∴ The angle between the planes VAB and 2
VCD is 36.9. Consider △BCD.

20. (a) Consider △FAP and △HCQ. BD 2  BC 2  CD 2 (Pyth. theorem)


AFP  CHQ  90 (given)
BD  6  6 cm
2 2
AP  CQ (given)
 6 2 cm
AF  CH (given)
1
∴ △FAP  △HCQ (RHS) ND  BD
∴ FP = HQ 2
PE = QE  3 2 cm
Consider △PQE.
NR  ND  RD
PQ 2  PE 2  QE 2 (Pyth. theorem)  ND  EM
3 cm  2QE
2 2 2  3
  3 2   cm
3  2
QE  cm
2 3(2 2  1)
 cm
HQ  HE  QE 2
MR = ED = 6 cm
 3  Consider △MNR.
  6   cm
 2 
Consider △HCQ. MR
tan MNR 
NR
QC 2  HQ 2  HC 2 (Pyth. theorem) 6 cm
2

 3  3(2 2  1)
QC  6    6 2 cm cm
 2 
  2
 7.14 cm (cor. to 3 sig. fig.) MNR  65.4 (cor. to 3 sig. fig.)
∴ The angle between the planes APQC and
(b) ABCD is 65.4.

Exercise 9G (p. 216)


Level 1
1. (a) Consider △ACB.
AB
cos ABC 
BC
AB
cos 30 
100 m
Let M AB  100 cos 30 m
and N be the mid-points of PQ and AC Consider △TBA.
respectively. Join MN, ND and ME. Draw a
TA
perpendicular PR  ND. tan TBA 
The angle between the planes APQC and ABCD AB
is MND. TA
∵ M is the mid-point of PQ. tan 20 
100 cos 30 m
∴ EM  PQ
3 TA  100 cos 30 tan 20 m
MQ  cm  31.5 m (cor. to 3 sig. fig.)
2
∴ The height of the tower TA is 31.5 m.

58
Certificate Mathematics in Action Full Solutions 4B

(b) Consider △ACB. TA


AC tan TCA 
sin ABC  AC
BC 500 m
AC tan 40 
sin 30  AC
100 m 500
AC  100 sin 30 m AC  m
tan 40
 50 m  596 m (cor. to 3 sig. fig.)
Consider △TCA.

TA
tan TCA 
AC
100 cos 30 tan 20

50
TCA  32.2 (cor. to 3 sig. fig.)
∴ The angle of elevation of T from C is
32.2.

2. (a) Consider △TAO.


TO
tan TAO 
AO
TO
tan 40 
50 m
TO  50 tan 40 m
 42.0 m (cor. to 3 sig. fig.)
∴ The height of the tower TO is 42.0 m.

(b) Consider △OAB.


OB
tan BAO 
OA
OB
tan 50 
50 m
OB  50 tan 50 m
 59.6 m (cor. to 3 sig. fig.)
∴ The distance between B and O is 59.6 m.

(c) Consider △TOB.

TO
tan TBO 
OB
50 tan 40

50 tan 50
TBO  35.1 (cor. to 3 sig. fig.)
∴ The angle of elevation of T from B is
35.1.

3. (a) TCA = 40


TBA = 60
Consider △TAC.

59
9 Basic Properties of CirclesMore about Trigonometry (II)

Consider △TAB. XC
TA tan XAC 
tan TBA  AC
AB 50
500 m 
tan 60  200 sin 70
AB
sin 74
500
AB  m XAC  14.3 (cor. to 3 sig. fig.)
tan 60
 289 m (cor. to 3 sig. fig.)
Consider △ACB.

BC 2  AB 2  AC 2 (Pyth. theorem)
2 2
 500   500 
BC      m
 tan 60    tan 40 
 662 m (cor. to 3 sig. fig.)

(b)

With the notations in the figure, consider


△ACB.
AC 2  BC 2  AB 2
cos ACB 
2( AC )( BC )
595.88 2  662.12 2  288.68 2

2(595.88)(662.12)
ACB  25.849
BCP  90  ACB
 90  25.849
 64.2 (cor. to 3 sig. fig.)
∴ The compass bearing of B from C is
N64.2W.

ACB  CAB  CBA  180


4. (a) (
ACB  36  70  180
ACB  74
sum of △)

By the sine formula,


AC AB

sin CBA sin ACB
AC 200 m

sin 70 sin 74
200 sin 70
AC  m
sin 74
Consider △XAC.

60
Certificate Mathematics in Action Full Solutions 4B

∴ The angle of elevation of X from A is LH


14.3. tan LBH 
HB
(b) Consider △ABC. 150 tan 25
By the sine formula, 
300 sin 60
BC AB LBH  15.1 (cor. to 3 sig. fig.)

sin CAB sin ACB ∴ The angle of elevation of L from B is
BC 200 m 15.1.

sin 36 sin 74
200 sin 36
BC  m
sin 74
Consider △XBC.

XC
tan XBC 
BC
50

200 sin 36
sin 74
XBC  22.2 (cor. to 3 sig. fig.)
∵ The angle of depression of B from X
= the angle of elevation of X from B
∴ The angle of depression of B from X is
22.2.

5. (a) Consider △ABH.


AH
cos BAH 
AB
AH
cos 60 
300 m
AH  300 cos 60 m
 150 m
Consider △LAH.
LH
tan LAH 
AH
LH
tan 25 
150 m
LH  150 tan 25 m
 69.9 m (cor. to 3 sig. fig.)
∴ The height of the lighthouse LH is 69.9 m.

(b) Consider △ABH.


HB
sin BAH 
AB
HB
sin 60 
300 m
HB  300 sin 60 m
 260 m (cor. to 3 sig. fig.)

(c) Consider △LHB.

61
9 Basic Properties of CirclesMore about Trigonometry (II)

6. (a) Consider △OAB.


BA
tan AOB 
OA
1000 m
tan 30 
OA
1000
OA  m
tan 30
 1000 3 m
Consider △ODC.
CD
tan DOC 
OD
1000 m
tan 20 
OD
1000
OD  m
tan 20
Consider △AOD.

OA
cos AOD 
OD
1000
tan 30

1000
tan 20
AOD  50.9 (cor. to 3 sig. fig.)
∴ The compass bearing of C from O is
N50.9E.

(b) Distance travelled by the helicopter


= BC
= 20v m
Consider △AOD.

OD 2  OA 2  AD 2 (Pyth. theorem)
 OA  BC
2 2

2 2
 1000   1000 
     (20v) 2
 tan 20   tan 30  
2
1000 1000 2
400v 2  
tan 2 20 tan 2 30
 1 1 
v  2500  
 tan 20 tan 30 
2 2

 107 (cor. to 3 sig. fig.)

7. (a)

Join AB.

62
Certificate Mathematics in Action Full Solutions 4B

Consider △TAC. AC AB
TC 
tan TAC  sin CBA sin ACB
AC 50
50 m m
tan 30  tan 30 75.931 m
AC 
sin CBA sin 30
50
AC  m 50 sin 30
tan 30
Consider △TBC. sin CBA  tan 30
75.931
CBA  34.769
TC
tan TBC  Reflex ABF  270  CBD  CBA
BC  270  30  34.769

50 m  335 (cor. to 3 sig. fig.)


tan 20  ∴ The true bearing of A from B is 335.
BC 8. (a) Consider △TAO.

50 tan TAO 
TO
BC  m OA
tan 20 tan 30 
20 m
ACB  120  90 OA
 30 20
OA  m
Consider △ACB. tan 30
By the cosine formula, Consider △OAC.
AB 2
 AC 2
 BC 2 AC  AB  BC
 2( AC )( BC ) cos ACB

  (30  15) m
2 2
 50  50 
     
 tan 30   tan 2 0 

5
AB m
2
 50 

45 m
0 
 cos 3 0
 tan 30  tan 20 
 75.9 m (cor. to 3 sig. fig.)

∴ The distance between A and B is 75.9 m. OA


cos OAC 
AC
(b)
20
 tan 30
45
OAC  39.7 (cor. to 3 sig. fig.)
∴ The true bearing of C from A is 39.7.

(b) Consider △OAC.

AC 2  OA2  OC 2 (Pyth. theorem)


2
With the notations in the figure,  20 
OC  452    m
 tan 30 
CBD  ACB (alt. s, DB // CA)  825 m
 30 Consider △TCO.
Consider △ACB.
By the sine formula, TO
tan TCO 
OC
20

825
TCO  34.8 (cor. to 3 sig. fig.)
∴ The angle of elevation of T from C is
34.8.

(c) Consider △OAB.

63
9 Basic Properties of CirclesMore about Trigonometry (II)

By the cosine formula,

OB 2
 OA 2  AB 2  2(OA)( AB ) cos OAB
2
 20 
   30 2 
 tan 30  
OB  m
 20 
2  (30 ) cos 39.664
 tan 30 
 500 m
Consider △TBO.

TO
tan TBO 
OB
20

500
TBO  41.8 (cor. to 3 sig. fig.)
∴ The angle of elevation of T from B is
41.8.

64
Certificate Mathematics in Action Full Solutions 4B

Level 2 TA
9. (a) tan TCA 
CA
TA

BA
 tan TBA
∴ TCA  TBA
 50
∵ The angle of depression of C from T
= the angle of elevation of T from C
∴ The angle of depression of C from T is
50.
With the notations in the figure,
ABC  70  30  40 10. (a)
BAD  FBA
 30
DAC  180  110 
(alt. s. AD With the notations in the figure, when the man
 70 is at A, let the length of his shadow be x cm.
BAC  BAD  DAC ∵ △GAF ~ △TOF (AAA)
 30  70 AF GA

 100 OF TO
// FB) ∴ x 1 .5

Consider △ABC. 40  x 14
ABC  BAC  ACB  180 14 x  60  1.5 x
40  100  ACB  180 x  4 .8
ACB  40 ∴ The length of the shadow of the man is
4.8 m when he is at A.
(sum of △)
(b)
By the sine formula,
BA BC

sin ACB sin BAC
BA 50 m

sin 40 sin 100
50 sin 40
BA  m
sin 100
 32.6 m (cor. to 3 sig. fig.)
With the notations in the figure,
HDC = 28
(b) Consider △TBA. Consider △HCD.
TA HC
tan TBA  tan HDC 
BA CD
TA 1.5 m
tan 50  tan 28 
50 sin 40 CD
m
sin 100 1.5
50 sin 40 tan 50 CD  m
TA  m tan 28
sin 100  2.82 m (cor. to 3 sig. fig.)
 38.9 m (cor. to 3 sig. fig.) ∴ The length of his shadow CD is 2.82 m.

(c) Consider △ABC. (c)


∵ ACB = ABC = 40
∴ CA = BA

65
9 Basic Properties of CirclesMore about Trigonometry (II)

HJ
tan HKJ 
Let J be a point on AB such that OJ  BA. When JK
the man is at J, his shadow is the shortest. HJ
With the notations in the figure, tan 15 
120 m
BA 2  OA 2  OB 2
HJ  120 tan 15 m
BA  40 2  25 2 m (Pyth. theorem)
HA  HJ  JA
 2225 m  (120 tan 15  25) m
 5 89 m  57.2 m (cor. to 3 sig. fig.)
Consider △OAJ and △BAO.

OAJ  BAO (common angle)


OJA  BOA  90
AOJ  180  OAJ  OJA
 180  BAO  BOA
 ABO
∴ △OAJ ~ △BAO (AAA)
OJ OA

BO BA
∴ OJ 40

25 m 5 89
200
OJ  m
89
Consider △LJK and △TOK.
∵ △LJK ~ △TOK (AAA)
JK LJ

OK TO
y 1.5

200
 y 14
∴ 89
300
14 y   1.5 y
89
24
y
89
 2.54 (cor. to 3 sig. fig.)
∴ The shortest length of his shadow is
2.54 m during his walk from A to B.

11. (a)

Draw a perpendicular from


K to HA at J.
JK  AB  120 m
JA  KB  25 m
HKJ = 15
Consider △HKJ.

66
Certificate Mathematics in Action Full Solutions 4B

(b) Consider △KPB.


KB
tan KPB 
BP
25 m
tan 20 
BP
25
BP  m
tan 20
ABP  90  50
 40
Consider △APB.
By the cosine formula,

AP 2  AB 2  BP 2  2( AB )( BP ) cos ABP
2
 25 
120   2
 
AP   tan 20  m
 25 
2(120)  cos 4 0
 tan 20 
 80.559 m
Consider △HPA.

HA
tan HPA 
AP
120 tan 15  25

80.559
HPA  35.4 (cor. to 3 sig. fig.)
∴ The angle of elevation of H from P is
35.4.

(c)

With the notations in the figure, consider


△APB.
By the cosine formula,
AB 2  AP 2  BP 2
cos BAP 
2( AB)( AP)
2
 25 
120  80.559  
2 2

  tan 20 
2(120)(80.559)
BAP  33.234
LAP  90  BAP
 90  33.234
 56.8 (cor. to 3 sig. fig.)
∴ The compass bearing of P from A is
S56.8E.

67
9 Basic Properties of CirclesMore about Trigonometry (II)

12. (a) Consider △TAG. PO


TG tan PAO 
tan TAG  OA
AG hm
hm tan 30 
tan 25  OA
AG h
OA  m
h tan 30
AG  m
tan 25  3hm
Consider △TBG.
TG
tan TBG 
BG
hm
tan 15 
BG
h
BG  m
tan 15
(b) Consider △ABG.
AG
tan ABG 
BG
h
 tan 25
h
tan 15
tan 15

tan 25
ABG  29.882
AG
sin ABG 
AB
h
sin 29.882  tan 25
450
h  105 (cor. to 3 sig. fig.)

(c)

With the notations in the figure,


HAB  ABG
 29.9 (cor. to 3 sig. fig.)
∴ The compass bearing of B from A is
S29.9E.

13. (a) Consider △PAO.

68
Certificate Mathematics in Action Full Solutions 4B

Consider △PBO.
PO
tan PBO 
OB
hm
tan 60 
OB
With the notations in the figure,
h cos OAB  cos OAC
OB  m ∵
tan 60 h 2  1350 h 2  5000
 
h 3 h  45 3 h 100 3 h
 m  or m
3  3  ∴ 20 h 2  27 000  9 h 2  45 000
 
Consider △PCO. 11 h 2  18 000
PO h  40.5 (cor. to 3 sig. fig.)
tan PCO 
OC
OA  3 hm
hm
tan 45   70.065 m
OC
OC  h m
h
OC  m  40.452 m
tan 45 Consider △OAC.
h m By the cosine formula,
OA 2  OC 2  AC 2
(b) (i) Consider △OAB. cos AOC 
By the cosine formula, 2(OA)(OC )
OA2  AB 2  OB 2 70.065 2  40.452 2  100 2
cos OAB  
2(OA)( AB) 2(70.065)(40.452)
 h 
2
AOC  127.548
( 3 h)  60   
2 2
DOC  180  AOC
  3  180  127.548
2( 3 h)(60)  52.5 (cor. to 3 sig. fig.)
h2 ∴ The compass bearing of C from O is
3h 2  3600  N52.5E.
 3
120 3 h 14. (a) Consider △XAC.
h  1350 
2
3 (h 2  1350)  XC
 or tan XAC 
45 3 h  135h  AC

XC
tan 35 
AC  AB  BC 71 m
(ii) XC  71 tan 35 m
 (60  40) m
 100 m  49.7 m (cor. to 3 sig. fig.)
Consider △OAC. ∴ The height of the tower XC is 49.7 m.
By the cosine formula,
OA2  AC 2  OC 2 (b) CAB  85  38
cos OAC   47
2(OA)( AC )
Consider △CAB.
( 3 h) 2  100 2  h 2 By the cosine formula,

2( 3 h)(100)
BC 2  AB 2  AC 2  2( AB )( AC ) cos CAB
3 h  10 000  h 2
2
 BC  702  712  2(70)(71) cos 47 m
200 3 h
 56.231 m
h 2  5000  3 (h 2  5000)  Consider △XBC.
 or
100 3 h  300h 

XC
tan XBC 
(c) BC
71 tan 35

56.231
XBC  41.5 (cor. to 3 sig. fig.)

69
9 Basic Properties of CirclesMore about Trigonometry (II)

∴ The angle of elevation of X from B is (c)


41.5.

With the notations in the figure,

BAF  90  85


 5
DCA  CAF (alt. s, DC // AF )
 CAB  BAF
 47  5
 52
Consider △CAB.
AC 2  BC 2  AB 2
cos ACB 
2( AC )( BC )
712  56.2312  70 2

2(71)(56.231)
ACB  65.566
GCB  270  DCA  ACB
 270  52  65.566
 152 (cor. to 3 sig. fig.)
∴ The true bearing of B from C is 152.

Revision Exercise 9 (p. 221)


Level 1
1. (a) Area of △ABC

1 
   30  24  sin 56  cm 2
2 
 298 cm (cor. to 3 sig. fig.)
2

(b) Area of △ABC

1 
   3.7  6  sin 102  cm 2
2 
 10.9 cm (cor. to 3 sig. fig.)
2

abc
(c) Let s  , where a = 5.1 cm, b = 7.4
2
cm and c = 4.1 cm.
5.1  7.4  4.1
∴ s cm
2
 8.3 cm
s – a = (8.3 – 5.1) cm = 3.2 cm
s – b = (8.3 – 7.4) cm = 0.9 cm
s – c = (8.3 – 4.1) cm = 4.2 cm
∴ Area of △ABC

70
Certificate Mathematics in Action Full Solutions 4B

 s ( s  a )( s  b)( s  c )  s ( s  a )( s  b)( s  c )
 8.3  3.2  0.9  4.2 cm 2
 86  44  36  6 cm 2
 10.0 cm 2 (cor. to 3 sig. fig.)  24 1419 cm 2
1
abc Area of △ABC   BC  AD
(d) Let s  , where a = 31 cm, b = 29 2
2 1
cm and c = 20 cm. 24 1419 cm 2   42 cm  h
2
31  29  20
∴ s cm 24 1419  2
2 h cm
42
 40 cm
 43.1 cm (cor. to 3 sig. fig.)
s – a = (40 – 31) cm = 9 cm
s – b = (40 – 29) cm = 11 cm
s – c = (40 – 20) cm = 20 cm 3. (a) Consider △AEF.
∴ Area of △ABC AE 2  AF 2  FE 2
(Pyth. theorem)
 s ( s  a )( s  b )( s  c ) AE  4 2  32 cm
 5 cm
 40  9  11  20 cm 2
Consider △AGF.
 281 cm 2 (cor. to 3 sig. fig.)
AG 2  AF 2  FG 2
(Pyth. theorem)
abc AG  4 2  6 2 cm
2. (a) Let s  , where a = 35 cm, b = 31
2  52 cm
cm and c = 24 cm.
 7.21 cm (cor. to 3 sig. fig.)
35  31  24

s cm Consider △GEH.
2
EG 2  EH 2  HG 2
 45 cm (Pyth. theorem)
s – a = (45 – 35) cm = 10 cm EG  6 2  3 2 cm
s – b = (45 – 31) cm = 14 cm
 45 cm
s – c = (45 – 24) cm = 21 cm
Area of △ABC
 6.71 cm (cor. to 3 sig. fig.)
 s ( s  a )( s  b)( s  c )
AE  AG  EG
 45  10  14  21 cm 2 (b) Let s  .
2
 210 3 cm 2
1 5  52  45
Area of △ABC   BC  AD ∴ s 2
cm
2
 9.460 cm
1
210 3 cm 2   35 cm  h s – AE = (9.460 – 5) cm = 4.460 cm
2 s – AG = (9.460 – 52 ) cm = 2.249 cm
210 3  2 s – EG = (9.460 – 45 ) cm = 2.752 cm
h cm
35 Area of △AEG
 20.8 cm (cor. to 3 sig. fig.)
 s ( s  AE )( s  AG )( s  EG )
abc  9.460  4.460  2.249  2.752 cm 2
(b) Let s  , where a = 42 cm, b = 50
2  16.2 cm 2 (cor. to 3 sig. fig.)
cm and c = 80 cm.
42  50  80 ADB  ADC  180
∴ s cm 4. (adj. s on st. line)
2 ADB  65  180
 86 cm ADB  115 
s – a = (86 – 42) cm = 44 cm Area of △ABC
s – b = (86 – 50) cm = 36 cm
s – c = (86 – 80) cm = 6 cm
 area of △ABD + area of △ADC
Area of △ABC

71
9 Basic Properties of CirclesMore about Trigonometry (II)

1 1
  AD  BD  sin ADB   AD  DC  sin ADC
2 2
1 1 
   12  8  sin 115    12  5  sin 65  cm 2
2 2 
 70.7 cm 2 (cor. to 3 sig. fig.)

5. (a) BC = AD = 20 cm (opp. sides of // gram)


Consider △ABC.
By the sine formula,

BC AB

sin BAC sin ACB
20 cm 16 cm

sin BAC sin 44
20 sin 44
sin BAC 
16
BAC  60.264 or 119.736 (rejected)
ABC  ACB  BAC  180 (
ABC  44  60.264  180
ABC  75.736
sum of △)

72
Certificate Mathematics in Action Full Solutions 4B

Area of ABCD b a
 2  area of △ABC 
sin B sin A
1 5 cm 10 cm
 2  AB  BC  sin ABC 
2 sin  sin 120
 1  5 sin 120
  2   16  20  sin 75.736  cm 2 sin  
 2  10
 310 cm 2 (cor. to 3 sig. fig.)   25.7 (cor. to 3 sig. fig.) or
154 (cor. to 3 sig. fig.) (rejected)
(b)
A  B  C  180 ( sum of
120  25.659  C  180
C  34.341
△)

By the sine formula,


Join BD.
DC = AB = 11 cm c a

Area of ABCD sin C sin A
 2  area of △BCD x cm 10 cm
1 
 2  BC  DC  sin BCD sin 34.341 sin 120
2 10 sin 34.341
 1  x
  2   15  11  sin 125  cm 2 sin 120
 2   6.51 (cor. to 3 sig. fig.)
 135 cm 2 (cor. to 3 sig. fig.)
(c) By the cosine formula,
1 a 2  b 2  c 2  2bc cos A
6. Area of △ABC   AB  BC  sin B
2 x 5 2  4 2  2(5)( 4) cos 60
1  21
5   x  ( x  1)  sin 30
2  4.58 (cor. to 3 sig. fig.)
x  x  20  0
2
a2  b2  c2
( x  4)( x  5)  0 cos C 
2ab
x  4 or x  5 (rejected)
( 21 ) 2  5 2  4 2
cos  
7. (a) By the cosine formula, 2( 21 )(5)
a2  c2  b2 3
cos B  
2ac 21
8.9  5.2 2  5 2
2   49.1 (cor. to 3 sig. fig.)
cos  
2(8.9)(5.2) (d) By the sine formula,
  28.6 (cor. to 3 sig. fig.) c a

sin C sin A
b2  c2  a2
cos A  2.8 cm 3.2 cm
2bc 
sin  sin 66
5 2  5.2 2  8.9 2 2.8 sin 66
cos  
2(5)(5.2) sin  
3.2
  122 (cor. to 3 sig. fig.)
  53.1 (cor. to 3 sig. fig.) or
(b) By the sine formula, 127 (cor. to 3 sig. fig.) (rejected)
A  B  C  180 ( sum of △)
66  B  53.068  180
B  60.932
By the sine formula,

73
9 Basic Properties of CirclesMore about Trigonometry (II)

b a

sin B sin A
x cm 3.2 cm

sin 60.932 sin 66
3.2 sin 60.932
x
sin 66
 3.06 (cor. to 3 sig. fig.)

A  B  C  180
8. (a) 42  B  68  180 ( sum of △)
B  70

74
Certificate Mathematics in Action Full Solutions 4B

By the sine formula, (d) By the sine formula,


c a
 a b
sin C sin A 
c 15 cm sin A sin B
 22 cm 26 cm
sin 68 sin 42 
15 sin 68 sin A sin 120
c cm 22 sin 120
sin 42 sin A 
 20.8 cm (cor. to 3 sig. fig.) 26
A  47.1 (cor. to 3 sig. fig.) or
b a
 133 (cor. to 3 sig. fig.) (rejected)
sin B sin A
A  B  C  180
b 15 cm
 47.121  120  C  180
sin 70 sin 42
C  12.9 (cor. to 3 sig. fig.)
15 sin 70
b cm By the sine formula,
sin 42
c b
 21.1 cm (cor. to 3 sig. fig.) 
sin C sin B
A  B  C  180 c 26 cm

(b) A  38  100  180 ( sum of △) sin 12.879 sin 120
A  42 26 sin 12.879
c cm
By the sine formula, sin 120
a b  6.69 cm (cor. to 3 sig. fig.)

sin A sin B
a 28 cm (e) By the cosine formula,
 b2  c2  a2
sin 42 sin 38 cos A 
28 sin 42 2bc
a cm
sin 38 35  20 2  30 2
2

 30.4 cm (cor. to 3 sig. fig.) 2(35)(20)
c b 29
 
sin C sin B 56
c 28 cm A  58.8 (cor. to 3 sig. fig.)

sin 100 sin 38 a2  c2  b2
cos B 
28 sin 100 2ac
c cm
sin 38 30 2  20 2  35 2

 44.8 cm (cor. to 3 sig. fig.) 2(30)(20)
1
(c) By the cosine formula, 
16
c 2  a 2  b 2  2ab cos C B  86.4 (cor. to 3 sig. fig.)
c 12 2  18 2  2(12)(18) cos 54 cm A  B  C  180 ( sum of △)
 14.6 cm (cor. to 3 sig. fig.) 58.811  86.417  C  180
C  34.8 (cor. to 3 sig. fig.)
b2  c2  a2
cos A 
2bc 9. (a) Consider △ACD.
18  14.6312  12 2
2 By the cosine formula,

2(18)(14.631)
A  41.6 (cor. to 3 sig. fig.)
A  B  C  180 ( sum of △)
41.569  B  54  180
B  84.4 (cor. to 3 sig. fig.)

75

( sum of △)
9 Basic Properties of CirclesMore about Trigonometry (II)

AC 2  CD 2  AD 2
cos ACD 
2( AC )(CD )
92  42  62

2(9)(4)
61

72
ACD  32.1 (cor. to 3 sig. fig.)

ACB  ACD  180


(b) (adj. s on
ACB  32.089  180
ACB  147.911 
st. line)

76
Certificate Mathematics in Action Full Solutions 4B

Consider △ABC. AK KB
By the sine formula, 
sin ABK sin KAB
AC AB AK 5 cm
 
sin ABD sin ACB sin 30 sin 40
9 cm 13 cm 5 sin 30
 AK  cm
sin ABD sin 147.911 sin 40
9 sin 147.911
sin ABD 
13
ABD  21.6 (cor. to 3 sig. fig.) or
158 (cor. to 3 sig. fig.) (rejected)

10. Consider △ABC.


By the cosine formula,

AC 2  AB 2  BC 2  2( AB )( BC ) cos ABC
AC  12 2  18 2  2(12)(18) cos 60 cm
 252 cm
 15.9 cm (cor. to 3 sig. fig.)
∵ ABCD is a parallelogram.
∴ DC = AB = 12 cm and BCD = 180 – 60 =
120
Consider △BCD.
By the cosine formula,

BD 2  BC 2  DC 2  2( BC )( DC ) cos BCD
BD  18 2  12 2  2(18)(12) cos 120 cm
 684 cm
 26.2 cm (cor. to 3 sig. fig.)

11. (a) Consider △BCK.


AKB  BCK  KBC (ext.  of
 30  80
 110 
△)

Consider △ABK.
By the sine formula,
AB KB

sin AKB sin KAB
AB 5 cm

sin 110  sin 40
5 sin 110 
AB  cm
sin 40
 7.31 cm (cor. to 3 sig. fig.)

(b) Consider △ABK.


KAB  ABK  AKB  180
40  ABK  110   180
ABK  30
( sum of △)

By the sine formula,

77
9 Basic Properties of CirclesMore about Trigonometry (II)

Consider △AKD.  AB  BC
AKB  ADK  DAK
(ext.  400 sin 102 400 sin 54 
110   ADK  75   m
 sin 24 sin 24 
ADK  35  1760 m (cor. to 3 sig. fig.)
 of △)

By the sine formula, 13. ADB  60  25


KD AK  35

sin DAK sin ADK DAB  90  60
5 sin 30  30
cm
KD
 sin 40
sin 75 sin 35
5 sin 30 sin 75
KD  cm
sin 35 sin 40
 6.55 cm (cor. to 3 sig. fig.)

12.

With the notations in the figure,


ABD  CAB (alt. s, DB // AC)
 54
ABC  CBD  ABD
 78  54
 24
Consider △ACB.
CAB  ACB  ABC  180
(
54  ACB  24  180
ACB  102
sum of △)

By the sine formula,


AB AC

sin ACB sin ABC
AB 400 m

sin 102 sin 24
400 sin 102
AB  m
sin 24
BC AC

sin CAB sin ABC
BC 400 m

sin 54 sin 24
400 sin 54
BC  m
sin 24
Total distance ran by the man

78
Certificate Mathematics in Action Full Solutions 4B

Consider △ADB. Distance travelled by boy X = (5  2) km = 10 km


By the sine formula, Distance travelled by boy Y = (6  2) km = 12 km
AB DB Hence, at 2 p.m., boys X and Y reach points G
 and H respectively, as shown in the figure.
sin ADB sin DAB
GOH  180  60
AB 9m
  120
sin 35 sin 30
OG  10 km
9 sin 35
AB  m OH  2 km
sin 30 Consider △OGH.
 10.3 m (cor. to 3 sig. fig.) By the cosine formula,
∴ The height of the flagstaff is 10.3 m.
GH 2  OG 2  OH 2  2(OG )(OH ) cos GOH
14.
GH  10 2  2 2  2(10)(2) cos 120 km
 124 km
 11.1 km (cor. to 3 sig. fig.)
∴ The distance between the two boys at 2 p.m.
is 11.1 km.
With the notations in the figure,
ACB  FAC (alt. s, BC // AF) (b)
 42
Consider △ACB.
AB
sin ACB 
AC
50 m
sin 42 
AC
50
AC  m
sin 42
With the notations in the figure, by the cosine
DAC  FAC  FAD formula,
 42  20 OG 2  GH 2  OH 2
 22 cos OGH 
2(OG )(GH )
ADC  FAD (alt. s, AF // CD)
10 2  ( 124 ) 2  2 2
 20 
Consider △ACD. 2(10)( 124 )
By the sine formula, 11
CD AC 
 124
sin DAC sin ADC
50 OGH  8.948
m
CD sin 42 KGO  90  60

sin 22 sin 20  30
50 sin 22 KGH  KGO  OGH
CD  m
sin 20 sin 42  30  8.948
 81.8 m (cor. to 3 sig. fig.)  38.9 (cor. to 3 sig. fig.)
∴ The distance between the two landmarks is ∴ The true bearing of Y from X at 2 p.m. is
81.8 m. 038.9.

15. (a) 16. (a)

79
9 Basic Properties of CirclesMore about Trigonometry (II)

With the notations in the figure, DE


ABF  DAB (alt. s, BF // DA) sin DCE 
DC
 60 DE
CBF  180  GBC (adj. s on
sin 10 
40 cm
 180  135
DE  40 sin 10 cm
 45
st. line)

ABC  ABF  CBF


 60  45
 105
Consider △ACB.
By the cosine formula,

AC 2  AB 2  BC 2  2( AB)( BC ) cos ABC


AC  4 2  5 2  2( 4)(5) cos 105 km
 7.17 km (cor. to 3 sig. fig.)

AB 2  AC 2  BC 2
cos BAC 
2( AB)( AC )
(b)
4 2  7.166 2  5 2

2( 4)(7.166)
BAC  42.374

DAB  BAC  HAC  180 (adj. s on st. line)


60  42.374  HAC  180
HAC  77.6 (cor. to 3 sig. fig.)
∴ The compass bearing of C from A is
S77.6E.

17. (a) The angle between BD and BA is ABD.


Consider △ABD.

AD
tan ABD 
AB
50

40
ABD  51.3 (cor. to 3 sig. fig.)
∴ The angle between BD and BA is 51.3.

(b) The angle between BD and plane BCEF is


DBE.
Consider △ABD.
AB
cos ABD 
BD
40 cm
cos 51.340 
BD
40
BD  cm
cos 51.340
Consider △DCE.

80
Certificate Mathematics in Action Full Solutions 4B

Consider △DBE. 1
PN  EF
DE 2
sin DBE  1 
BD    16  cm
40 sin 10  2 
  8 cm
40
Consider △PMN.
cos 51.340
DBE  6.23 (cor. to 3 sig. fig.) PN
∴ The angle between BD and plane BCEF is tan PMN 
NM
6.23.
8

18. (a) 5
PMN  58.0 (cor. to 3 sig. fig.)
∴ The angle between the planes PAB and
ABFG is 58.0.

19. (a) Consider △ABC.


ABC  ACB  BAC  180
60  72  BAC  180
Join AC and BD. BAC  48
AQ is the projection of AP on plane ABCD. ( sum of △)
The angle that PA makes with the base ABCD is
PAQ. By the sine formula,
Consider △ABC. AB BC
AC 2  AB 2  BC 2 
(Pyth. theorem) sin ACB sin BAC
AC  12 2  16 2 cm AB 14 cm

 20 cm sin 72 sin 48
1 14 sin 72
AQ  AC AB  cm
2 (property of rectangle) sin 48
1   17.9 cm (cor. to 3 sig. fig.)
   20  cm
2  Consider △PBA.
 10 cm PA
tan PBA 
PQ = GA = 5 cm AB
Consider △PAQ. PA
tan 30 
PQ 14 sin 72
tan PAQ  cm
AQ sin 48
14 sin 72 tan 30
5 PA  cm
 sin 48
10
PAQ  26.6 (cor. to 3 sig. fig.)  10.3 cm (cor. to 3 sig. fig.)
∴ The angle that PA makes with the base
ABCD is 26.6. (b) Area of △ABC
1
  AB  BC  sin ABC
(b) 2
1 
   17.917  14  sin 60  cm 2
 2 
 109 cm 2 (cor. to 3 sig. fig.)

(c) Volume of the tetrahedron PBCA


1
Join PA   area of △ABC  PA
and PB. 3
Let M and N be the mid-points of AB and GF
respectively.
The angle between the planes PAB and ABFG is
PMN.

81
9 Basic Properties of CirclesMore about Trigonometry (II)

1  20. (a) Consider △TBP.


   108.616  10.344  cm 3 TP
3  tan TBP 
 375 cm (cor. to 3 sig. fig.)
3 BP
45 m
tan 40 
BP
45
BP  m
tan 40
Consider △TAP.
TP
tan TAP 
AP
45 m
tan 30 
AP
45
AP  m
tan 30
BPA  200  150
 50
Consider △BAP.
By the cosine formula,

AB 2
 BP 2
 AP 2
 2
2
 45 
  
 tan 40 
AB 
 45 
2 
 tan 4 0 
 59.8 m (co r. to

(b)

With the notations in the figure,


BPQ = 200 – 180 = 20
RBP  BPQ (alt. s, RB // PQ)
 20
Consider △BAP.
By the cosine formula,
BP 2  AB 2  AP 2
cos PBA 
2( BP)( AB)
2 2
 45   45 
   59.8112   
tan 40   tan 30 

 45 
2 (59.811)
 tan 40 
PBA  86.618
RBA  RBP  PBA
 20  86.618
 107 (cor. to 3 sig. fig.)
∴ The true bearing of A from B is 107.

82
Certificate Mathematics in Action Full Solutions 4B

22. Since A is an obtuse angle, cos A < 0.


21. (a) By the cosine formula,
b2  c2  a2
cos A 
2bc
20  c 2  30 2
2

2( 20)(c)
c 2  500

40c
cos A  0

With the notations in the figure, c 2  500
AG = BH = 800 m 0
Consider △ACG. 40c
∵ c>0
AG
tan ACG  c 2  500  0
CG ∴  c 2  500
800 m
tan 50  0  c  500
CG
∴ A possible integral value of c is 15 cm. (or any
800 other reasonable answers)
CG  m
tan 50
Consider △CHG. 23.
GH
tan HCG 
CG
GH
tan 70 
800
m
tan 50 Assume the required triangle is ABC.
800 tan 70 ∵ Area of △ABC = 24 cm2
GH  m
tan 50 Take C = 45, we have
∴ The speed of the aeroplane 1
ab sin C  24 cm 2
800 tan 70 2
m
 tan 50 1
ab sin 45  24 cm 2
5s 2
 369 m/s (cor to 3 sig. fig.) 1  1 
ab   24 cm 2
2  2 
(b) Consider △CHG.
CG ab  48 2 cm 2
cos HCG  Take a = 8 cm, then
CH
800 48 2
b cm
m 8
cos 70  tan 50
CH  6 2 cm
By the cosine formula,
800
CH  m
tan 50 cos 70 c 2  a 2  b 2  2ab cos C
Consider △BCH.
c 8 2  (6 2 ) 2  2(8)(6 2 ) cos 45 cm
BH  2 10 cm
tan BCH 
CH a  c  [8 2  (2 10 ) 2 ] cm 2
2 2

800
  104 cm 2
800
b 2  (6 2 ) 2 cm 2
tan 50 cos 70
 72 cm 2
BCH  22.2 (cor. to 3 sig. fig.)
∴ a + c2  b2
2

∴ The angle of elevation of B from C is ∴ B  90


22.2. i.e. △ABC is not a right-angled triangle.
∴ The lengths of the sides of a triangle which
satisfies the required two conditions are 8 cm,

83
9 Basic Properties of CirclesMore about Trigonometry (II)

6 2 cm and 2 10 cm. (or any other AB 2  BC 2  AC 2


reasonable answers) cos ABC 
2( AB )( BC )
Level 2
82  7 2  62
24. (a) Consider △ACD. 
2(8)(7)
CAD  ACD  ADC ( 180
sum of △)
11
45  ACD  90  180 
16
ACD  45
Consider △ABC.
BAC  ABC  ACD (ext.  of )

BAC  30  45
BAC  15
By the sine formula,
AC BC

sin ABC sin BAC
AC 25 cm

sin 30 sin 15
25 sin 30
AC  cm
sin 15
Consider △ACD.
AD
cos CAD 
AC
x cm
cos 45 
25 sin 30
cm
sin 15
25 sin 30
x  cos 45 
sin 15
 34.2 (cor. to 3 sig. fig.)

(b) Consider △ACD.


ADC  DAC  ACD  180 (
ADC  40  68  180
ADC  72
sum of △)

By the sine formula,


AC AD

sin ADC sin ACD
AC 7 cm

sin 72 sin 68
7 sin 72
AC  cm
sin 68
Consider △ABC.
By the cosine formula,

BC 2
 AB 2
 AC 2
 2( AB )( AC ) cos BAC
2
 7 sin 72 
8 2
   
 sin 68 
BC  cm
 7 sin 72  
2(8)  cos 80
 sin 68 
x  9.78 (cor. to 3 sig. fig.)

(c) Consider △ABC.


By the cosine formula,

84
Certificate Mathematics in Action Full Solutions 4B

Consider △ABD. 26. Let OC = x cm.


BD = (7 + 5) cm = 12 cm Area of the shaded region
By the cosine formula, = area of sector OAB  area of △AOC

AD 2  AB 2  BD 2  2( AB )( BD ) cos ABD 50 1


    252    25  x  sin 50  cm 2
 11   360 2 
AD  8 2  12 2  2(8)(12)  cm
 16  ∵ Area of the shaded region = 160 cm2

x 76 50 1
  252    25  x  sin 50  160
 8.72 (cor. to 3 sig. fig.) 360 2
x = 11.8 (cor. to 3 sig. fig.)
(d) Consider △AMC. ∴ OC = 11 .8 cm
AC = AM = 8 cm (given)
ACM = AMC (base s, isos. △) 27. (a) Consider △OAC.
MAC  AMC  ACM  180 ( By the cosine formula,
40  2ACM  180
ACM  70 OA 2  OC 2  AC 2
cos AOC 
sum of △) 2(OA)(OC )
6 2  9 2  132
By the cosine formula, 
2(6)( 9)
MC 2  AM 2  AC 2  2( AM )( AC ) cos MAC 13
 
MC  8 2  8 2  2(8)(8) cos 40 cm 27
 5.472 cm
AOC  119  (cor. to 3 sig. fig.)
BM  MC  5.472 cm
BC  BM  MC OA 2  AC 2  OC 2
cos OAC 
 (5.472  5.472) cm 2(OA)( AC )
 10.944 cm 6 2  132  9 2
Consider △ABC. 
2(6)(13)
By the cosine formula,
31

AB 2  AC 2  BC 2  2( AC )( BC ) cos ACB 39
AB  8 2  10.944 2  2(8)(10.944) cos 70 cm OAC  37.4 (cor. to 3 sig. fig.)
x  11.1 (cor. to 3 sig. fig.)
(b)
25. Consider △AOC.
OAC  AOC  ACO  180 (
30  50  ACO  180
ACO  100
sum of △)

By the sine formula, Join OB.


OC OA ∵ AC is the tangent to the circle at B.
 ∴ OB  AC
sin OAC sin ACO Consider △OAB.
OC 13 cm OB
 sin OAB 
sin 30 sin 100 OA
13 sin 30 OB
OC  cm sin 37.357 
sin 100 6 cm
Area of the shaded region OB  6 sin 37.357 cm
= area of sector OAB  area of △AOC
 3.64 cm (cor. to 3 sig. fig.)
 50 1 13 sin 30 
    13 2    13   sin 50(c) cm 2
 360 2 sin 100 
 40.9 cm (cor. to 3 sig. fig.)
2

85
9 Basic Properties of CirclesMore about Trigonometry (II)

28. (a)

With the notations in the figure,


total area of the shaded regions
= area of △OAC  area of sector ODE
Join AC.
1 Consider △ABC.
   6  9  sin 118.782 20 
2 AC 2  AB 2  BC 2  2( AB )( BC ) cos ABC
118 .782 20 
  3.640 66 2   cm
2
AC  9 2  8 2  2(9)(8) cos 60 cm
360 
 73 cm
 9.93 cm 2 (cor. to 3 sig. fig.)
 8.54 cm (cor. to 3 sig. fig.)

(b) Consider △ABC.


AB  BC  AC
Let s1 = .
2
9  8  8.544
∴ s1 = cm
2
= 12.772 cm
s1  AB = (12.772  9) cm = 3.772 cm
s1  BC = (12.772  8) cm = 4.772 cm
s1  AC = (12.772  8.544) cm = 4.228 cm
Consider △ACD.
AC  CD  DA
Let s2 = .
2
8.544  7  6
∴ s2 = cm
2
= 10.772 cm
s2  AC = (10.772  8.544) cm = 2.228 cm
s2  CD = (10.772  7) cm = 3.772 cm
s2  DA = (10.772  6) cm = 4.772 cm
Area of ABCD
= area of △ABC + area of △ACD

 s1 ( s1  AB )( s1  BC )( s1  AC ) 
s 2 ( s 2  AC )( s 2  CD )( s 2  DA)
 ( 12.772  3.772  4.772  4.228 
10.772  2.228  3.772  4.772 ) cm 2
 52.0 cm 2 (cor. to 3 sig. fig.)

29. (a) ABC  35  25  60


ACB  30  20  50
BAC  ABC  ACB  180
BAC  60  50  180
BAC  70
( sum of △)

Consider △ABC.
By the sine formula,

86
Certificate Mathematics in Action Full Solutions 4B

AB BC

sin ACB sin BAC
AB 12 cm

sin 50 sin 70
12 sin 50
AB  cm
sin 70
 9.78 cm (cor. to 3 sig. fig.)

87
9 Basic Properties of CirclesMore about Trigonometry (II)

(b) Consider △BCD. ∵ Area of △ABC = area of △ADC


BDC  DBC  BCD  180 ( sum
1 1
BDC  25  20  180  AB  BC  sin ABC   AD 
2 2
BDC  135 CD  sin AD
of △) 1 1
 2  6  sin    x 4
2 2
By the sine formula, sin(180   )
BD BC 6 sin   2 x sin 
 x  3
sin BCD sin BDC
AD  3 cm
BD 12 cm ∴

sin 20 sin 135 (b) Consider △ABC.
12 sin 20
BD  cm AC 2  AB 2  BC 2  2( AB )( BC ) cos ABC
sin 135
Consider △ABD. AC  2 2  6 2  2( 2)( 6) cos  cm
AD 2
 AB 2
 BD 2
Consider
2( △ACD.
AB )( BD ) cos A
2 2
 12 sin 50   12 sin 20 
 2  2  
AD   sin 70  2  AC
sin 135   AD  CD  2( AD)(CD) cos ADC
 12 sin 50  12 sin 20 
2   cos 3
sin 70 2 sin 135


6.03 cm
2

(cor. to 3 sig. fig. )
AC 

3  4  2(3)(4) cos (180   ) cm
∴ 2 2  6 2  2( 2)(6) cos 
30. Consider △ABD.
By the sine formula,
BD AB  3 2  4 2  2(3)( 4) cos (180   )
 40  24 cos   25  24 cos 
sin BAD sin ADB
BD sin ADB 48 cos   15
sin BAD  5
AB cos  
Consider △ADC. 16
By the sine formula,
DC AC
 sin   1  cos 2
sin DAC sin ADC
2
DC sin ADC 5
 1   
sin DAC 
AC (adj. s on st. line) (c)  16 
ADB  ADC  180 231
ADB  180  ADC

256
sin ADB  sin (180  ADC ) 231
 sin ADC

16
∵ AD is the angle bisector of BAC. Area of quadrilateral ABCD
∴ = area of △ABC + area of △ADC
= 2  area of △ABC
BAD  DAC 1
sin BAD  sin DAC  2  AB  BC  sin ABC
2
BD sin ADB DC sin ADC  1 231 
   2  2 6  cm 2
AB AC  2 16 
 
BD DC
  11 .4 cm (cor. to 3 sig. fig.)
2
AB AC
BD AB 32. Consider △PBA.

DC AC BPA  60  50
 10
31. (a) Let AD = x cm.
By the sine formula,
∵ ABCD is a cyclic quadrilateral.
∴ ABC  ADC  180(opp. s, cyclic quad.)
ADC  180  

88
Certificate Mathematics in Action Full Solutions 4B

AP BA

sin PBA sin BPA
AP 100 m

sin 50 sin 10
100 sin 50
AP  m
sin 10

89
9 Basic Properties of CirclesMore about Trigonometry (II)

Consider △AQP. PQ
PAQ  60  40 sin PCQ 
PC
 20
DE
APQ  90  60 
DE
 30
sin 20 cos 50
PAQ  AQP  APQ  180 
(  sin 20 cos 50
20  AQP  30  180
AQP  130
PCQ  12.7 (cor. to 3 sig. fig.)
∴ The angle between CP and plane BCEF is
sum of △)
12.7.
By the sine formula,
PQ AP

sin PAQ sin AQP
100 sin 50
m
PQ
 sin 10
sin 20 sin 130
 100 sin 50 
PQ    sin 20  m
 sin 10 sin 130 
 197 m (cor. to 3 sig. fig.)
∴ The height of the church is 197 m.

33. (a) Consider △DCE.


DE
sin DCE 
CD
DE
sin 20 
CD
DE
CD 
sin 20
(b)

Draw PQ  FE. Join QC.


The angle between CP and plane BCEF is
PCQ.
Consider △PCD.
CD
cos PCD 
PC
DE
cos 50  sin 20
PC
DE
PC 
sin 20 cos 50
PQ = DE
Consider △PCQ.

90
Certificate Mathematics in Action Full Solutions 4B

34. (a) Consider △ABD and △ABC.


AB = AB (common side)
ABD  ABC  90
BD  BC
( given)
(given)
∴ △ABD  △ABC (SAS)
AD = AC
∵ AD = AC and DE = EC = 3 cm
∴ AE  DC
∵ BD = BC and DE = EC = 3 cm
∴ BE  DC
With the notations in the figure, consider Consider △ABD.
△AFB. AD 2  AB 2  BD 2
AB (Pyth. theorem)
tan AFB  AD  10 2  8 2 cm
FB
4m  2 41 cm
tan 60  Consider △ADE.
FB
AD 2  AE 2  DE 2
4 (Pyth.
FB  m AE  ( 2 41 ) 2  32 cm
tan 60
 2.31 m (cor. to 3 sig. fig.)  155 cm
theorem)
GBF  30
CBF  90  GBF Consider △BDE.
 90  30 BD 2  BE 2  DE 2
(Pyth. theorem)
 60 BE  8 2  3 2 cm
Consider △BFC.  55 cm
By the cosine formula,
Consider △ABE.
FC 2
 BC 2
 FB 2  2( BC )( FB ) cos CBF
2 BE

10  

2 4 

 tan 60 
 2(10 ) 

4
cos AEB 


 tan 60 
FC
cos 60
AE m

 9.07 m (cor. to 3 sig. fig.) 55



155
(b) Consider △BFC.
By the cosine formula, 11

31
FB 2  FC 2  BC 2 AEB  53.4 (cor. to 3 sig. fig.)
cos BFC 
2( FB )( FC ) ∴ The angle between the planes ACD and
2.3094 2  9.0686 2  10 2 BCD is 53.4.

2( 2.3094)(9.0686) (b)
BFC  107 (cor. to 3 sig. fig.)

35. (a)

Let F be the
mid-point of MN.
The angle between the planes ANM and BNM is
AFB.
∵ M and N are the mid-points of AD and AC
Let E be the mid-point of DC. Join AE and BE. respectively.
The angle between the planes ACD and BCD is ∴ AM = MD and AN = NC
AEB. ∵ AD = AC

91
9 Basic Properties of CirclesMore about Trigonometry (II)

∴ AM = AN Consider △AFN and △AEC.


∵ AM = AN and MF = FN FAN  EAC (common angles)
∴ AF  MN AFN  AEC  90
ANF  180  FAN  AFN
 180  EAC  AEC
 ACE
∴ △AFN ~ △AEC (AAA)
AF AN

AE AC
AF 1

AE 2
1
AF  AE
2
155
 cm
2
155
FE  cm
2
Consider △BEF.
2
BF  BE 2
 FE 2
 2( BE ) ( FE )
2
 1 55 
( 55 ) 2
   
 2 
BF   
 15 5  11 
2( 55 )  
 2  31 
  
1 55
 cm
2

Consider △ABF.

AF 2  BF 2  AB 2
cos AFB 
2( AF )( BF )
2 2
 155   155 
    10
2

2 2
    
 155  155 
2  
 2  2 
9
 
31
AFB  107 (cor. to 3 sig. fig.)
∴ The angle between the planes ANM and
BNM is 107.

36. (a)

Draw XG  AD.
GX = AB
= 15 m
YG = (30  10  10) m
= 10 m
Consider △YGX.

92
Certificate Mathematics in Action Full Solutions 4B

XY 2  GX 2  YG 2 Consider △ABD.
(Pyth. theorem)
AB 2  AD 2  BD 2
XY  152  10 2 m
AD  7 2  52 m
 5 13 m
The total distance travelled by the man  24 m (Pyth. theorem)
 CX  XY  YA AF  AP  FP
 (10  5 13  10) m  AP  CR
 38.0 m (cor. to 3 sig. fig.)  (5  4) m
(b) 1m
Consider △ADF.

AF
sin ADF 
AD
1m

24 m
ADF  11 .8 (cor. to 3 sig. fig.)
With the notations in the figure,
JX = 10 m ∴ The angle that the sheet makes with the
JXK = 25 horizontal ground is 11.8.
Consider △JXK.
(b)
JK
sin JXK 
JX
JK
sin 25 
10 m
JK  10 sin 25 m
YL  JK
 10 sin 25 m
The angle between the path XY and the
horizontal ground is YXL.
Consider △YXL. With the notations in the figure, consider
△ABF.
YL AB 2  AF 2  BF 2
(Pyth. theorem)
sin YXL 
XY BF  7 2  12 m
10 sin 25  48 m

5 13 Similarly, CF = BF = 48 m
YXL  13.6 (cor. to 3 sig. fig.) Consider △BFC.
∴ The angle between the path XY and the BF 2  CF 2  BC 2
horizontal ground is 13.6.
cos BFC 
2( BF )(CF )
37. (a) ( 48 ) 2  ( 48 ) 2  102

2( 48 )( 48 )
1

24
sin BFC  1  cos2 BFC
2
1 
 1    
 24 
5 23

Let D be the mid-point of BC. With the 24
notations in the figure, the angle that the sheet
makes with the horizontal ground is ADF. Area of the shadow of the sheet at noon
∵ AB = AC = 7 m and BD = DC = 5 m  area of △BFC
∴ AD  BC

93
9 Basic Properties of CirclesMore about Trigonometry (II)

1 YH  GH  GR  RY
  BF  CF  sin BFC
2  5  4 
1 5 23  2  5  10  m
   48  48  m  tan   tan  
 2 24 
 1 
 24.0 m 2 (cor. to 3 sig. fig.)   5 m
 tan  
HZ  YZ  YH
(c) (i)
  1 
 10    5  m
  tan  
 1 
 5  m
 tan  

With the notations in the figure, consider


△ADF.
AD 2  AF 2  DF 2

DF  ( 24 ) 2  1 m
 23 m
GP  HX (Pyth. theorem)
 DF
 23 m
GH  PX
RP  CF
 48 m
Consider △GPR.
RP 2  GR 2  GP 2

GR  ( 48 ) 2  ( 23 ) 2 m
5m
(Pyth. theorem)

Consider △CRZ.
CR
tan  
RZ
4
RZ  m
tan 
RY  RZ  YZ
 4 
  10  m
 tan  
Consider △APX.
AP
tan  
PX
5
PX  m
tan 

94
(Pyth. theorem)

Certificate Mathematics in Action Full Solutions 4B

Consider △XHY. CE
tan CAE 
YX 2  YH 2  HX 2 EA
2 6 cm
YX  
1
 5   ( 23 ) 2 m 
2.7362 cm
 tan  
2
CAE  65.5 (cor. to 3 sig. fig.)
1
 23    5  m ∴ The angle between AC and the horizontal
 tan   ground is 65.5.
Consider △XHZ. (Pyth. theorem)
(b) Consider △CAE.
ZX 2  HZ 2  HX 2
CE
 1 
2 sin CAE 
ZX  5    ( 23 ) 2 m AC
 tan   6 cm
sin 65.485 
 1 
2
AC
 23   5   m
 tan  6
AC  cm
sin 65.485
When YX = ZX, Consider △ACB.
By the cosine formula,
2 2
1 1 
23    5   23   5   cos CBA 
AB 2  BC 2  AC 2
 tan    tan  
2( AB )( BC )
1 1
55  6 
2
tan  tan  5 5 2 2

2   sin 65.485 
0 2(5)(5)
tan 
  90 or 270 (rejected) CBA  82.5 (cor. to 3 sig. fig.)
∴ The shape of the shadow of the sheet
is not an isosceles triangle. 39. (a)
(ii) The area of the shadow is equal to the area
of △PQR. It is because the bases and the
heights of the two shadows are the same.

38. (a) The angle between AC and the horizontal


ground is CAE.
∵ ADB = AEC = 90
∴ BD // CE (corr. s equal)
∵ BD // CE and AB = BC = 5 cm
∴ AD = DE (intercept theorem)
Consider △AEC. With the notation
AC 2  AE 2  CE 2 s in the figure, let HQ = x m.
(Pyth. theorem)
Then QK = (600  x) m.
AE  10  6 cm
2 2
Consider △HPQ.
 8 cm HQ
AD  DE tan HPQ 
PQ
1 xm
 AE tan 65 
∴ 2 PQ
1
   8  cm PQ 
x
m
2  tan 65
 4 cm
Consider △DAE. Consider △KPQ.
By the cosine formula,

EA 2  AD 2  DE 2  2( AD )( DE ) cos ADE
EA  4 2  4 2  2( 4)(4) cos 40 cm
 2.7362 cm
Consider △CAE.

95
9 Basic Properties of CirclesMore about Trigonometry (II)

QK Consider △HPQ.
tan KPQ  HQ
PQ sin HPQ 
PH
(600  x ) m
tan 40  431.258 m
PQ sin 65 
PH
600  x 431.258
PQ  m PH  m
tan 40 sin 65
x 600  x  476 m (cor. to 3 sig. fig.)

tan 65 tan 40
Consider △KPQ.
x tan 40  600 tan 65  x tan 65
QK
600 tan 65 sin KPQ 
x  PK
tan 40  tan 65 168.742 m
 431.258 sin 40 
PK
600  x  168.742 168.742
PK  m
sin 40
 263 m (cor. to 3 sig. fig.)

(b) Consider △APH.


AH
tan APH 
PH
AH
tan 44 
475.841 m
AH  475.841 tan 44 m
 460 m (cor. to 3 sig. fig.)
Consider △BPK.
BK
tan BPK 
PK
BK
tan 33 
262.516 m
BK  262.516 tan 33 m
 170 m (cor. to 3 sig. fig.)

(c)

With the notations in the figure,


RB = HK = 600 m

AR  AH  RH
 AH  BK
 ( 475.841 tan 44  262.516 tan 33) m
 289.034 m
Consider △ABR.

96
Certificate Mathematics in Action Full Solutions 4B

AR OC 2  OB 2  BC 2
tan ABR  cos COB 
RB 2(OC )(OB )
289.034 m
 ( 3h) 2  90 2  h 2
600 m 
2( 3h)(90)
ABR  25.7 (cor. to 3 sig. fig.)
∵ The angle of depression of B from A 2h  8100
2

= the angle of elevation of A from B 180 3h
∴ The angle of depression of B and A is
25.7. h 2  4050

90 3h
40. (a) Consider △DOC.
DC ∵ cos COA  cos COB
tan DOC  (c)
OC h 2  1350 h 2  4050
hm 
tan 30  45 3h 90 3h
OC ∴ 2h 2  2700  h 2  4050
h
OC  m h 2  1350
tan 30
h  36.7 (cor. to 3 sig. fig.)
 3h m
Consider △DAC.
(d)
DC
tan DAC 
AC h 2  1350
cos COA 
hm 45 3h
tan 60 
AC 1350  1350

h 45 3 ( 1350 )
AC  m
tan 60 COA  19.5 (cor. to 3 sig. fig.)
h The compass bearing of B from O is
 m ∴
3 N19.5E.
Consider △COA.
41. (a) The angle between DF and BF is DFB.
OC 2  OA2  AC 2 Consider △BFG.
cos COA 
2(OC )(OA) BF 2  BG 2  FG 2 (Pyth. theorem)
2
h BF  6  8 cm
2 2

( 3h ) 2  602   
  3  10 cm
Consider △DFE.
2( 3h )(60)
DF 2  DE 2  EF 2 (Pyth. theorem)
2
h
3h  3600 
2
DF  6 2  8 2 cm
 3
 10 cm
120 3h Consider △DAB.
h 2  1350 DB 2  DA2  AB 2
 (Pyth. theorem)
45 3h DB  8 2  82 cm
 128 cm
(b) Consider △DBC.
Consider △DFB.
DC By the cosine formula,
tan DBC 
BC
hm BF 2  DF 2  DB 2
tan 45  cos DFB 
BC 2( BF )( DF )
h 10 2  10 2  ( 128 ) 2
BC  m 
tan 45 2(10)(10)
hm 9
Consider △COB. 
25
DFB  68.9 (cor. to 3 sig. fig.)
∴ The angle between DF and BF is 68.9.

97
9 Basic Properties of CirclesMore about Trigonometry (II)

(b)

Let EG and FH intersects at N. Join FM and


MN.
The angle between FM and plane EFGH is
MFN.
Consider △FGH.
FH 2  FG 2  GH 2
(Pyth. theorem)
FH  8 2  8 2 cm
 8 2 cm

98
Certificate Mathematics in Action Full Solutions 4B

∵ EFGH is a square.
BC 2  CD 2  BD 2
FN  NH cos BCD 
2( BC )(CD )
1
 FH 8 2  6 2  12 2
∴ 2 (property of 
2(8)(6)
1
   8 2  cm 11
2   
24
 4 2 cm
square)
BCD  117 .280

MN = CH = 6 cm ABC  BCD  180


Consider △MFN. ABC  117.280  180
ABC  63 (cor. to the nearest degree)
MN
tan MFN 
FN 2. Answer: C
6 cm A  B  C  180 ( sum of △)

4 2 cm A  60  75  180
MFN  46.7 (cor. to 3 sig. fig.) A  45
∴ The angle between FM and plane EFGH is By the sine formula,
46.7. AC BC

sin B sin A
(c)
AC BC

sin 60 sin 45
AC sin 60

BC sin 45
3
 2
1
Join CM. 2
∵ DFB is an isosceles triangle. 6
∴ FM  DB 
∵ CDB is an isosceles triangle. 2
∴ CM  DB
The angle between the planes FDB and DBC is 3. Answer : C
FMC. By the cosine formula,
Consider △MFN.
AB 2  BC 2  AC 2
FMN  MFN  FNM  ( 180  of △)
sum cos ABC 
2( AB )( BC )
FMN  46.686  90  180
FMN  43.314
(3x ) 2  x 2  7 2
cos 60 
CMN  90 2(3x )( x )
FMC  FMN  CMN 1 10 x 2  49

 43.314  90 2 6x 2
 133 (cor. to 3 sig. fig.) 3x 2  10 x 2  49
∴ The angle between the planes FDB and x2  7
DBC is 133.
x  7
Multiple Choice Questions (p. 229)
1. Answer: B
DC = AB = 6 cm (opp. sides of // gram) 4. Answer : D
Consider △BCD.

99
9 Basic Properties of CirclesMore about Trigonometry (II)

By the sine formula,


With the notations in the figure, AC AB
by the sine formula, 
sin ABC sin ACB
PR PQ AC 12 m
 
sin PQR sin PRQ sin 60 sin 55
30 m 96 m 12 sin 60
 AC  m
sin PQR sin 100 sin 55
30 sin 100 3
sin PQR  12 
96  2 m
PQR  17.924 or 162.076 (rejected) sin 55
RPQ  PQR  PRQ  180 6 3
( sum  m
RPQ  17.924  100  180 sin 55
RPQ  62.076
of △) 7. Answer : D
Consider △BDC.
TPQ  TPR  RPQ By the cosine formula,
 55  62.076
BC 2  BD 2  CD 2  2( BD )(CD ) cos BDC
 117 .1
∴ The true bearing of Q from P is 117.1. BC  7 2  ( 3 2 ) 2  2(7)( 3 2 ) cos 45 cm
 5 cm
5. Answer : B
Consider △ABC.
AB  AC  BC
Let s = .
2

With the notations in the figure,


by the cosine formula,
AB 2  AC 2  BC 2
cos BAC 
2( AB )( AC )
42  72  92

2( 4)( 7)
2
 
7
BAC  106.6
FAC  180  DAB  BAC
 180  36  106.6
 37 (cor. to the nearest degree)
∴ The compass bearing of C from A is S37W.

6. Answer : A
ABC  90  30  60
BAC  90  25  65
Consider △ABC.
ACB  BAC  ABC  180 ( sum
ACB  65  60  180
ACB  55
of △)

100
Certificate Mathematics in Action Full Solutions 4B

745
∴ s= cm
2
= 8 cm Let N be the mid-point of DE.
s  AB = (8  7) cm = 1 cm Join MN and HN.
s  AC = (8  4) cm = 4 cm The angle between MH and plane CDEH is MHN.
s  BC = (8  5) cm = 3 cm MN  AD  4 cm
Area of △ABC
1
 s ( s  AB )( s  AC )( s  BC ) NE  DE
2
 8  1  4  3 cm 2
1
 4 6 cm 2    10  cm
2 
8. Answer: B  5 cm
Area of ABDE = area of △ACE  area of △BCD Consider △NHE.
HN 2  HE 2  NE 2
1 (Pyth. theorem)
  AC  CE  sin ACE HN  4 2  5 2 cm
2
1
 BC  CD  sin BCD  41 cm
2 Consider △MHN.
1
   ( 4  2)  (3  5)  MN
 2 tan MHN 
HN
1 
sin    4  3  sin   m
2
4 cm
2  
 18 sin  m2 41 cm
∵ Area of ABDE = 14 m2 MHN  32 (cor. to the nearest degree)
18 sin   14
∴ The angle between MH and plane CDEH is 32.
7
∴ sin  
9 11. Answer : C
  51 (cor. to the nearest AD = BC = 4 cm
Consider △ACD.
AC 2  AD 2  CD 2
degree) (Pyth. theorem)
CD  6 2  4 2 cm
9. Answer : C
Consider △ACD.  20 cm
The angle between CD and plane BCEF is DCE.
AD
cos CAD  Consider △DCE.
AC
x CE
cos   cos DCE 
AC CD
x 3 cm
AC  
cos  20 cm
Consider △ABC. DCE  48 (cor. to the nearest degree)
By the sine formula,
∴ The angle between CD and plane BCEF is 48.
BC AC

sin BAC sin ABC 12. Answer : A
x
BC cos 

sin  sin 
x sin 
BC 
sin  cos 

10. Answer : A
Let J be the mid-
point of BC.
Joint HJ and JK.

101
9 Basic Properties of CirclesMore about Trigonometry (II)

AK  KD Consider △HAK.
HA2  HK 2  AK 2
1
 AD HK  32  2 2 cm
2 (Pyth. theorem)
1  5 cm
   4  cm KJ  DC
2 
 2 cm  4 cm
The angle between the planes HBC and ABCD is
HJK.
Consider △HJK.

HK
tan HJK 
JK
5

4
HJK  29 (cor. to the nearest degree)
∴ The angle between the planes HBC and ABCD
is 29.

HKMO (p. 231)


1.

Join OA, OF, OB and OE.


Let OA = y cm,
then OA = OB = OE = OF = y cm.
360
AOF 
6
 60
Consider △AOF.
Area of △AOF

1
  OA  OF  sin AOF
2
1
 y 2 sin 60 cm 2
2
3y2
 cm 2
4
Area of ABCDEF = 6  area of △AOF
3y2
3 3  6
4
y 2
QP  BE
 2 y cm
 2 2 cm
Area of square PQRS  QP
2

 ( 2 2 ) 2 cm 2
∵ Area of square PQRS = X cm2

102
Certificate Mathematics in Action Full Solutions 4B

∴ X  (2 2 ) 2
8

103
9 Basic Properties of CirclesMore about Trigonometry (II)

2. By the cosine formula,


BD  181  180 cos 
c 2  a 2  b 2  2ab cos C
∴ 3
c 2  a 2  b 2  2ab cos 60  181  180  
 5
c 2  a 2  b 2  ab
 17
a 2  b 2  c 2  ab Consider △ABD.
a b AD  AB  BD
 P Let s1 = .
bc ac 2
a ( a  c)  b(b  c) 9  10  17
P ∴ s1 =
( a  c)(b  c) 2
= 18
a 2  ac  b 2  bc
P s1  AD  18  9  9
( a  c)(b  c)
∵ s1  AB  18  10  8
c 2  ab  ac  bc
P s1  BD  18  17  1
( a  c)(b  c)
Consider △BCD.
b( a  c)  c(a  c)
P BC  DC  BD
( a  c)(b  c) Let s2 = .
2
(a  c)(b  c) 10  21  17
P ∴ s2 =
(a  c)(b  c) 2
P 1 = 24
s2  BC = 24  10 = 14
s2  DC = 24  21 = 3
3.
s2  BD = 24  17 = 7
Area of the cyclic quadrilateral
= area of △ABD + area of △BCD
 s1 ( s1  AD )( s1  AB)( s1  BD) 
s 2 ( s 2  BC )( s 2  DC )( s 2  BD )
 18  9  8  1  24  14  3  7
 36  84
 120
Consider △ABD.
By the cosine formula,

BD 2  AD 2  AB 2  2( AD )( AB ) cos DAB
 9 2  10 2  2(9)(10) cos 
 181  180 cos 
BAD  BCD  180 (opp. s, cyclic
BCD  180  
quad.)
Consider △BCD.
By the cosine formula,

BD 2  BC 2  DC 2  2( BC )( DC ) cos BCD
 10 2  212  2(10)( 21) cos (180   )
 541  420 cos 
181  180 cos   541  420 cos 
∴ 600 cos   360
3
cos   
5

104

Vous aimerez peut-être aussi